What is the probability that the sum of digits of a random $k$-digit number is $n$?












12












$begingroup$


Let $X_1, X_2, dots, X_k$ each be random digits. That is, they are independent random variables each uniformly distributed over the finite set ${0, 1, 2, 3, 4, 5, 6, 7, 8, 9}$. Let $S = X_1 + X_2 + dots + X_k$. Given some large integer $n$, what is the probability that $S = n$?



When $n$ or $k$ is small, the exact number can be computed as



$$[z^n]left(frac{1-z^{10}}{1-z}right)^k = frac{1}{10^k} sum_{10r+s=n} (-1)^{r+s} binom{k}{r} binom{-k}{s} = frac{1}{10^k} sum_{r} (-1)^{r} binom{k}{r} binom{k+n-10r-1}{n-10r}$$



(see the long list of questions below to see derivations) but what I need is an asymptotic expression useful for large $n$ and $k$, and I don't know how to either derive one from this formula, or arrive at one independently.



(For now, I'm not worrying about the complication of insisting that $X_1$ be nonzero, though feel free to consider it if it actually helps.)





What I have tried, part 1: As the $X_i$s are IID random variables (with mean $mu = 4.5$ and variance $sigma^2 = 8.25$), the central limit theorem applies, so we expect $Pr(S = n)$ to be highest for $n$ around $4.5k$, and the probability distribution of $S$ to be bell-curve-shaped around that value (and most of the probability will be distributed for $n$ about $O(sqrt{k})$ from that value).



Trying to make this more precise, the central limit theorem gives us
$$sqrt{k}(S/k - 4.5) xrightarrow{d} N(0,8.25) quad text{i.e.} quad lim _{kto infty}Pr left[sqrt{k}(S_{k}/k- 4.5)le zright]= Phileft(frac {z}{sqrt{8.25}}right)$$
where $Phi(x) = frac12 left[1+operatorname{erf} left(frac{x}{sqrt {2}}right)right]$ is the CDF of the standard normal distribution $N(0,1)$ (and erf is a special function) and therefore
$$Pr(S le x) = Pr(S/sqrt{k} - 4.5sqrt{k} le x/sqrt{k} - 4.5sqrt{k}) to Phileft(frac{x - 4.5k}{sqrt{8.25k}}right)$$
and so, applying a continuity correction,
$$begin{align}Pr(S = n) &approx Pr(n - 0.5 < S le n + 0.5) \
&to Phileft(frac{n + 0.5 - 4.5k}{sqrt{8.25k}}right) - Phileft(frac{n - 0.5 - 4.5k}{sqrt{8.25k}}right) \
&= frac12operatorname{erf}left(frac{2n+1-9k}{sqrt{66k}}right) - frac12operatorname{erf}left(frac{2n-1-9k}{sqrt{66k}}right) \
&overset{?}{approx} frac{2}{sqrt{pi}}frac{1}{sqrt{66k}}expleft(-left(frac{2n-9k}{sqrt{66k}}right)^2right)
end{align}$$

but I neither know whether this is correct and rigorous, nor what to do with this next.





What I have tried, part 2: There are many questions on this site about calculating this number exactly:




  • Probability of random integer's digits summing to 12


  • how many integers between one and 100000 have the sum equal to fifteen?


  • How many numbers between 100 and 900 have sum of their digits equal to 15?


  • Stars and bars to find "how many $x$ digit numbers are there with sum of digits $y$"?


  • Find numbers whose sum of digits equals a value


  • Counting the numbers with certain sum of digits.


  • How many natural numbers are there less than $90000$ that have the sum of digits equal to $8$?


  • How many integers between [3,000, 8,000] have digit sum 20?


  • Counting $4$-digits numbers whose digits sum is $9$


  • How many numbers between $1$ and $9999$ have sum of their digits equal to $8$? $16$?


  • How many numbers between $0$ and $999,999$ are there whose digits sum to $r$


  • How many positive integers less than 1,000,000 have the sum of their digits equal to 19?


  • How many positive integers < 10^6 have sum of digits equal to 19


  • For how many integers from 1 through 99,999 is the sum of their digits equal to 10?


  • If I have an integer, how many numbers are there whose digits sum to the integer?


  • Determine the number of positive integer x where x≤9,999,999 and the sum of the digits in x equals 31.


  • Find number of positive integers less than $10^8$ with digit sum of $24$


  • Find the number of positive integers whose digits add up to 42


  • Enumerating number of solutions to an equation



The best that can be got from reading all of them is the exact formula mentioned at the top of the question (done with generating functions or inclusion-exclusion), not an asymptotic one. In particular I'd like to be able to get something which can be summed over $k$, to answer the following question:





Question 2: Let $X_{i,j}$ each be IID random digits as before, for $i = 1, 2, dots$ and $j = 1, 2, dots, i$. Let $S_k = X_{k,1} + X_{k,2} + dots + X_{k,k}$ be the sum of $k$ random digits. So we have an infinite sequence of random variables (sums) $S_1, S_2, S_3, dots$ each obtained by adding the digits of a random number of a different length. Given an integer $n$, what is the probability that some element of this sequence is exactly equal to $n$?



In other words, for each $k$ there is a probability distribution over $n$, and we want to know the total probability that falls on a single integer $n$. (Basically for each $n$ there will be some significant probability for $k$ around $n/4.5$ and the probability will fall off significantly for $k$ further away from this.)



(Again, feel free to add or remove the condition that $X_{k,1}$ is actually distributed over the set ${1, 2, dots, 9}$, i.e. is nonzero.)





What I have tried, part 3: I tried to read Distribution of the sum-of-digits function of random integers: A survey (which I found by searching for some relevant terms), and many of the papers it references. But I got pretty lost trying to figure out what is true for base-$2$ versus base-$10$, and things like that. Perhaps the answer to my question is buried somewhere in there, but I'm not sure.










share|cite|improve this question











$endgroup$












  • $begingroup$
    I would give you +100 if i could for thoroughness. Great question...
    $endgroup$
    – Eleven-Eleven
    Jan 7 at 17:23
















12












$begingroup$


Let $X_1, X_2, dots, X_k$ each be random digits. That is, they are independent random variables each uniformly distributed over the finite set ${0, 1, 2, 3, 4, 5, 6, 7, 8, 9}$. Let $S = X_1 + X_2 + dots + X_k$. Given some large integer $n$, what is the probability that $S = n$?



When $n$ or $k$ is small, the exact number can be computed as



$$[z^n]left(frac{1-z^{10}}{1-z}right)^k = frac{1}{10^k} sum_{10r+s=n} (-1)^{r+s} binom{k}{r} binom{-k}{s} = frac{1}{10^k} sum_{r} (-1)^{r} binom{k}{r} binom{k+n-10r-1}{n-10r}$$



(see the long list of questions below to see derivations) but what I need is an asymptotic expression useful for large $n$ and $k$, and I don't know how to either derive one from this formula, or arrive at one independently.



(For now, I'm not worrying about the complication of insisting that $X_1$ be nonzero, though feel free to consider it if it actually helps.)





What I have tried, part 1: As the $X_i$s are IID random variables (with mean $mu = 4.5$ and variance $sigma^2 = 8.25$), the central limit theorem applies, so we expect $Pr(S = n)$ to be highest for $n$ around $4.5k$, and the probability distribution of $S$ to be bell-curve-shaped around that value (and most of the probability will be distributed for $n$ about $O(sqrt{k})$ from that value).



Trying to make this more precise, the central limit theorem gives us
$$sqrt{k}(S/k - 4.5) xrightarrow{d} N(0,8.25) quad text{i.e.} quad lim _{kto infty}Pr left[sqrt{k}(S_{k}/k- 4.5)le zright]= Phileft(frac {z}{sqrt{8.25}}right)$$
where $Phi(x) = frac12 left[1+operatorname{erf} left(frac{x}{sqrt {2}}right)right]$ is the CDF of the standard normal distribution $N(0,1)$ (and erf is a special function) and therefore
$$Pr(S le x) = Pr(S/sqrt{k} - 4.5sqrt{k} le x/sqrt{k} - 4.5sqrt{k}) to Phileft(frac{x - 4.5k}{sqrt{8.25k}}right)$$
and so, applying a continuity correction,
$$begin{align}Pr(S = n) &approx Pr(n - 0.5 < S le n + 0.5) \
&to Phileft(frac{n + 0.5 - 4.5k}{sqrt{8.25k}}right) - Phileft(frac{n - 0.5 - 4.5k}{sqrt{8.25k}}right) \
&= frac12operatorname{erf}left(frac{2n+1-9k}{sqrt{66k}}right) - frac12operatorname{erf}left(frac{2n-1-9k}{sqrt{66k}}right) \
&overset{?}{approx} frac{2}{sqrt{pi}}frac{1}{sqrt{66k}}expleft(-left(frac{2n-9k}{sqrt{66k}}right)^2right)
end{align}$$

but I neither know whether this is correct and rigorous, nor what to do with this next.





What I have tried, part 2: There are many questions on this site about calculating this number exactly:




  • Probability of random integer's digits summing to 12


  • how many integers between one and 100000 have the sum equal to fifteen?


  • How many numbers between 100 and 900 have sum of their digits equal to 15?


  • Stars and bars to find "how many $x$ digit numbers are there with sum of digits $y$"?


  • Find numbers whose sum of digits equals a value


  • Counting the numbers with certain sum of digits.


  • How many natural numbers are there less than $90000$ that have the sum of digits equal to $8$?


  • How many integers between [3,000, 8,000] have digit sum 20?


  • Counting $4$-digits numbers whose digits sum is $9$


  • How many numbers between $1$ and $9999$ have sum of their digits equal to $8$? $16$?


  • How many numbers between $0$ and $999,999$ are there whose digits sum to $r$


  • How many positive integers less than 1,000,000 have the sum of their digits equal to 19?


  • How many positive integers < 10^6 have sum of digits equal to 19


  • For how many integers from 1 through 99,999 is the sum of their digits equal to 10?


  • If I have an integer, how many numbers are there whose digits sum to the integer?


  • Determine the number of positive integer x where x≤9,999,999 and the sum of the digits in x equals 31.


  • Find number of positive integers less than $10^8$ with digit sum of $24$


  • Find the number of positive integers whose digits add up to 42


  • Enumerating number of solutions to an equation



The best that can be got from reading all of them is the exact formula mentioned at the top of the question (done with generating functions or inclusion-exclusion), not an asymptotic one. In particular I'd like to be able to get something which can be summed over $k$, to answer the following question:





Question 2: Let $X_{i,j}$ each be IID random digits as before, for $i = 1, 2, dots$ and $j = 1, 2, dots, i$. Let $S_k = X_{k,1} + X_{k,2} + dots + X_{k,k}$ be the sum of $k$ random digits. So we have an infinite sequence of random variables (sums) $S_1, S_2, S_3, dots$ each obtained by adding the digits of a random number of a different length. Given an integer $n$, what is the probability that some element of this sequence is exactly equal to $n$?



In other words, for each $k$ there is a probability distribution over $n$, and we want to know the total probability that falls on a single integer $n$. (Basically for each $n$ there will be some significant probability for $k$ around $n/4.5$ and the probability will fall off significantly for $k$ further away from this.)



(Again, feel free to add or remove the condition that $X_{k,1}$ is actually distributed over the set ${1, 2, dots, 9}$, i.e. is nonzero.)





What I have tried, part 3: I tried to read Distribution of the sum-of-digits function of random integers: A survey (which I found by searching for some relevant terms), and many of the papers it references. But I got pretty lost trying to figure out what is true for base-$2$ versus base-$10$, and things like that. Perhaps the answer to my question is buried somewhere in there, but I'm not sure.










share|cite|improve this question











$endgroup$












  • $begingroup$
    I would give you +100 if i could for thoroughness. Great question...
    $endgroup$
    – Eleven-Eleven
    Jan 7 at 17:23














12












12








12


4



$begingroup$


Let $X_1, X_2, dots, X_k$ each be random digits. That is, they are independent random variables each uniformly distributed over the finite set ${0, 1, 2, 3, 4, 5, 6, 7, 8, 9}$. Let $S = X_1 + X_2 + dots + X_k$. Given some large integer $n$, what is the probability that $S = n$?



When $n$ or $k$ is small, the exact number can be computed as



$$[z^n]left(frac{1-z^{10}}{1-z}right)^k = frac{1}{10^k} sum_{10r+s=n} (-1)^{r+s} binom{k}{r} binom{-k}{s} = frac{1}{10^k} sum_{r} (-1)^{r} binom{k}{r} binom{k+n-10r-1}{n-10r}$$



(see the long list of questions below to see derivations) but what I need is an asymptotic expression useful for large $n$ and $k$, and I don't know how to either derive one from this formula, or arrive at one independently.



(For now, I'm not worrying about the complication of insisting that $X_1$ be nonzero, though feel free to consider it if it actually helps.)





What I have tried, part 1: As the $X_i$s are IID random variables (with mean $mu = 4.5$ and variance $sigma^2 = 8.25$), the central limit theorem applies, so we expect $Pr(S = n)$ to be highest for $n$ around $4.5k$, and the probability distribution of $S$ to be bell-curve-shaped around that value (and most of the probability will be distributed for $n$ about $O(sqrt{k})$ from that value).



Trying to make this more precise, the central limit theorem gives us
$$sqrt{k}(S/k - 4.5) xrightarrow{d} N(0,8.25) quad text{i.e.} quad lim _{kto infty}Pr left[sqrt{k}(S_{k}/k- 4.5)le zright]= Phileft(frac {z}{sqrt{8.25}}right)$$
where $Phi(x) = frac12 left[1+operatorname{erf} left(frac{x}{sqrt {2}}right)right]$ is the CDF of the standard normal distribution $N(0,1)$ (and erf is a special function) and therefore
$$Pr(S le x) = Pr(S/sqrt{k} - 4.5sqrt{k} le x/sqrt{k} - 4.5sqrt{k}) to Phileft(frac{x - 4.5k}{sqrt{8.25k}}right)$$
and so, applying a continuity correction,
$$begin{align}Pr(S = n) &approx Pr(n - 0.5 < S le n + 0.5) \
&to Phileft(frac{n + 0.5 - 4.5k}{sqrt{8.25k}}right) - Phileft(frac{n - 0.5 - 4.5k}{sqrt{8.25k}}right) \
&= frac12operatorname{erf}left(frac{2n+1-9k}{sqrt{66k}}right) - frac12operatorname{erf}left(frac{2n-1-9k}{sqrt{66k}}right) \
&overset{?}{approx} frac{2}{sqrt{pi}}frac{1}{sqrt{66k}}expleft(-left(frac{2n-9k}{sqrt{66k}}right)^2right)
end{align}$$

but I neither know whether this is correct and rigorous, nor what to do with this next.





What I have tried, part 2: There are many questions on this site about calculating this number exactly:




  • Probability of random integer's digits summing to 12


  • how many integers between one and 100000 have the sum equal to fifteen?


  • How many numbers between 100 and 900 have sum of their digits equal to 15?


  • Stars and bars to find "how many $x$ digit numbers are there with sum of digits $y$"?


  • Find numbers whose sum of digits equals a value


  • Counting the numbers with certain sum of digits.


  • How many natural numbers are there less than $90000$ that have the sum of digits equal to $8$?


  • How many integers between [3,000, 8,000] have digit sum 20?


  • Counting $4$-digits numbers whose digits sum is $9$


  • How many numbers between $1$ and $9999$ have sum of their digits equal to $8$? $16$?


  • How many numbers between $0$ and $999,999$ are there whose digits sum to $r$


  • How many positive integers less than 1,000,000 have the sum of their digits equal to 19?


  • How many positive integers < 10^6 have sum of digits equal to 19


  • For how many integers from 1 through 99,999 is the sum of their digits equal to 10?


  • If I have an integer, how many numbers are there whose digits sum to the integer?


  • Determine the number of positive integer x where x≤9,999,999 and the sum of the digits in x equals 31.


  • Find number of positive integers less than $10^8$ with digit sum of $24$


  • Find the number of positive integers whose digits add up to 42


  • Enumerating number of solutions to an equation



The best that can be got from reading all of them is the exact formula mentioned at the top of the question (done with generating functions or inclusion-exclusion), not an asymptotic one. In particular I'd like to be able to get something which can be summed over $k$, to answer the following question:





Question 2: Let $X_{i,j}$ each be IID random digits as before, for $i = 1, 2, dots$ and $j = 1, 2, dots, i$. Let $S_k = X_{k,1} + X_{k,2} + dots + X_{k,k}$ be the sum of $k$ random digits. So we have an infinite sequence of random variables (sums) $S_1, S_2, S_3, dots$ each obtained by adding the digits of a random number of a different length. Given an integer $n$, what is the probability that some element of this sequence is exactly equal to $n$?



In other words, for each $k$ there is a probability distribution over $n$, and we want to know the total probability that falls on a single integer $n$. (Basically for each $n$ there will be some significant probability for $k$ around $n/4.5$ and the probability will fall off significantly for $k$ further away from this.)



(Again, feel free to add or remove the condition that $X_{k,1}$ is actually distributed over the set ${1, 2, dots, 9}$, i.e. is nonzero.)





What I have tried, part 3: I tried to read Distribution of the sum-of-digits function of random integers: A survey (which I found by searching for some relevant terms), and many of the papers it references. But I got pretty lost trying to figure out what is true for base-$2$ versus base-$10$, and things like that. Perhaps the answer to my question is buried somewhere in there, but I'm not sure.










share|cite|improve this question











$endgroup$




Let $X_1, X_2, dots, X_k$ each be random digits. That is, they are independent random variables each uniformly distributed over the finite set ${0, 1, 2, 3, 4, 5, 6, 7, 8, 9}$. Let $S = X_1 + X_2 + dots + X_k$. Given some large integer $n$, what is the probability that $S = n$?



When $n$ or $k$ is small, the exact number can be computed as



$$[z^n]left(frac{1-z^{10}}{1-z}right)^k = frac{1}{10^k} sum_{10r+s=n} (-1)^{r+s} binom{k}{r} binom{-k}{s} = frac{1}{10^k} sum_{r} (-1)^{r} binom{k}{r} binom{k+n-10r-1}{n-10r}$$



(see the long list of questions below to see derivations) but what I need is an asymptotic expression useful for large $n$ and $k$, and I don't know how to either derive one from this formula, or arrive at one independently.



(For now, I'm not worrying about the complication of insisting that $X_1$ be nonzero, though feel free to consider it if it actually helps.)





What I have tried, part 1: As the $X_i$s are IID random variables (with mean $mu = 4.5$ and variance $sigma^2 = 8.25$), the central limit theorem applies, so we expect $Pr(S = n)$ to be highest for $n$ around $4.5k$, and the probability distribution of $S$ to be bell-curve-shaped around that value (and most of the probability will be distributed for $n$ about $O(sqrt{k})$ from that value).



Trying to make this more precise, the central limit theorem gives us
$$sqrt{k}(S/k - 4.5) xrightarrow{d} N(0,8.25) quad text{i.e.} quad lim _{kto infty}Pr left[sqrt{k}(S_{k}/k- 4.5)le zright]= Phileft(frac {z}{sqrt{8.25}}right)$$
where $Phi(x) = frac12 left[1+operatorname{erf} left(frac{x}{sqrt {2}}right)right]$ is the CDF of the standard normal distribution $N(0,1)$ (and erf is a special function) and therefore
$$Pr(S le x) = Pr(S/sqrt{k} - 4.5sqrt{k} le x/sqrt{k} - 4.5sqrt{k}) to Phileft(frac{x - 4.5k}{sqrt{8.25k}}right)$$
and so, applying a continuity correction,
$$begin{align}Pr(S = n) &approx Pr(n - 0.5 < S le n + 0.5) \
&to Phileft(frac{n + 0.5 - 4.5k}{sqrt{8.25k}}right) - Phileft(frac{n - 0.5 - 4.5k}{sqrt{8.25k}}right) \
&= frac12operatorname{erf}left(frac{2n+1-9k}{sqrt{66k}}right) - frac12operatorname{erf}left(frac{2n-1-9k}{sqrt{66k}}right) \
&overset{?}{approx} frac{2}{sqrt{pi}}frac{1}{sqrt{66k}}expleft(-left(frac{2n-9k}{sqrt{66k}}right)^2right)
end{align}$$

but I neither know whether this is correct and rigorous, nor what to do with this next.





What I have tried, part 2: There are many questions on this site about calculating this number exactly:




  • Probability of random integer's digits summing to 12


  • how many integers between one and 100000 have the sum equal to fifteen?


  • How many numbers between 100 and 900 have sum of their digits equal to 15?


  • Stars and bars to find "how many $x$ digit numbers are there with sum of digits $y$"?


  • Find numbers whose sum of digits equals a value


  • Counting the numbers with certain sum of digits.


  • How many natural numbers are there less than $90000$ that have the sum of digits equal to $8$?


  • How many integers between [3,000, 8,000] have digit sum 20?


  • Counting $4$-digits numbers whose digits sum is $9$


  • How many numbers between $1$ and $9999$ have sum of their digits equal to $8$? $16$?


  • How many numbers between $0$ and $999,999$ are there whose digits sum to $r$


  • How many positive integers less than 1,000,000 have the sum of their digits equal to 19?


  • How many positive integers < 10^6 have sum of digits equal to 19


  • For how many integers from 1 through 99,999 is the sum of their digits equal to 10?


  • If I have an integer, how many numbers are there whose digits sum to the integer?


  • Determine the number of positive integer x where x≤9,999,999 and the sum of the digits in x equals 31.


  • Find number of positive integers less than $10^8$ with digit sum of $24$


  • Find the number of positive integers whose digits add up to 42


  • Enumerating number of solutions to an equation



The best that can be got from reading all of them is the exact formula mentioned at the top of the question (done with generating functions or inclusion-exclusion), not an asymptotic one. In particular I'd like to be able to get something which can be summed over $k$, to answer the following question:





Question 2: Let $X_{i,j}$ each be IID random digits as before, for $i = 1, 2, dots$ and $j = 1, 2, dots, i$. Let $S_k = X_{k,1} + X_{k,2} + dots + X_{k,k}$ be the sum of $k$ random digits. So we have an infinite sequence of random variables (sums) $S_1, S_2, S_3, dots$ each obtained by adding the digits of a random number of a different length. Given an integer $n$, what is the probability that some element of this sequence is exactly equal to $n$?



In other words, for each $k$ there is a probability distribution over $n$, and we want to know the total probability that falls on a single integer $n$. (Basically for each $n$ there will be some significant probability for $k$ around $n/4.5$ and the probability will fall off significantly for $k$ further away from this.)



(Again, feel free to add or remove the condition that $X_{k,1}$ is actually distributed over the set ${1, 2, dots, 9}$, i.e. is nonzero.)





What I have tried, part 3: I tried to read Distribution of the sum-of-digits function of random integers: A survey (which I found by searching for some relevant terms), and many of the papers it references. But I got pretty lost trying to figure out what is true for base-$2$ versus base-$10$, and things like that. Perhaps the answer to my question is buried somewhere in there, but I'm not sure.







probability-theory asymptotics probability-limit-theorems






share|cite|improve this question















share|cite|improve this question













share|cite|improve this question




share|cite|improve this question








edited Jan 9 at 3:02







ShreevatsaR

















asked Jan 6 at 9:34









ShreevatsaRShreevatsaR

34.6k668107




34.6k668107












  • $begingroup$
    I would give you +100 if i could for thoroughness. Great question...
    $endgroup$
    – Eleven-Eleven
    Jan 7 at 17:23


















  • $begingroup$
    I would give you +100 if i could for thoroughness. Great question...
    $endgroup$
    – Eleven-Eleven
    Jan 7 at 17:23
















$begingroup$
I would give you +100 if i could for thoroughness. Great question...
$endgroup$
– Eleven-Eleven
Jan 7 at 17:23




$begingroup$
I would give you +100 if i could for thoroughness. Great question...
$endgroup$
– Eleven-Eleven
Jan 7 at 17:23










4 Answers
4






active

oldest

votes


















4












$begingroup$

Here we show the main term $frac{2}{sqrt{pi}}frac{1}{sqrt{66k}}$ in OPs asymptotic expansion is correct using the saddle-point method.




The coefficients in the expansion of the polynomials
begin{align*}
left(frac{1-z^{10}}{1-z}right)^{k}=left(1+z+z^2+z^3+z^4+z^5+z^6+z^7+z^8+z^9right)^{k}tag{1}
end{align*}

form a unimodal sequence. Taking even values $2k$ the maximum is the coefficient of the middle term
begin{align*}
[z^{9k}]left(frac{1-z^{10}}{1-z}right)^{2k}qquad kgeq 0tag{2}
end{align*}




The coefficients in (2) form a sequence starting with $(1,10,670, 55,252,4,816,030,ldots)$. These numbers are called Lucky ticket numbers and are stored in OEIS. In fact they are the even central decanomial coefficients which are the largest coefficients in the expansion of (1).




They also cite an asymptotic formula for the largest coefficient of $left(1+z+cdots+z^qright)^k$ which gives in the case (2)
begin{align*}
color{blue}{[z^{9k}]left(frac{1-z^{10}}{1-z}right)^{2k}sim 10^{2k}frac{1}{sqrt{33pi k}}}tag{3}
end{align*}

corresponding to OPs main term in his expansion ($k$ even).




We can prove the asymptotic expansion (3) using the saddle-point method. In order to do so we closely follow section VIII.8 Large Powers in Analytic Combinatorics by P. Flajolet and R. Sedgewick. We also give a little bit of surrounding information to ease readability.




VIII.8 Large powers:





  • (p. 585): The extraction of coefficients in powers of a fixed function and more generally in functions of the form $A(z)B(z)^k$ constitutes a prototypical and easy application of the saddle-point method. We will accordingly be concerned here with the problem of estimating
    begin{align*}
    [z^K]A(z)cdot B(z)^k=frac{1}{2pi i}oint A(z)B(z)^kfrac{dz}{z^{K+1}}
    end{align*}

    as both $k$ and $K$ get large.




In our situation (3) we have $A(z)=1$ and $B(z)=left(1+z+z^2+cdots+z^9right)^2$ with $K=9k$. What follows are conditions which need to be fulfilled by $A(z)$ and $B(z)$ in order to apply the saddle-point method.




VIII. 8.1. Large powers: saddle-point bounds: We consider throughout this section two fixed functions, $A(z)$ and $B(z)$ satisfying the following conditions.




  • L1: The functions $A(z)=sum_{jgeq 0}a_jz^j$ and $B(z)=sum_{jgeq 0}a_jz^j$ are analytic at $0$ and have non-negative coefficients; furthermore it is assumed (without loss of generality) that $B(0) ne 0$.


  • L2: The function $B(z)$ is aperiodic in the sense that $gcd{j|b_j>0}=1$. (Thus $B(z)$ is not a function of the form $beta(z^p)$ for some integer $pgeq 2$ and some $beta$ analytic at $0$.)


  • L3: Let $Rleq infty$ be the radius of convergence of $B(z)$; the radius of convergence of $A(z)$ is at least as large as $R$.





We observe conditions L1 to L3 are fulfilled for $A(z)=1$ and $B(z)=left(1+z+z^2+cdots+z^9right)^2$ with radius of convergence $R=infty$. In the following we need the quantity $T$ called spread which is defined as



begin{align*}
color{blue}{T}&:=lim_{zto R^{-}}frac{zB^{prime}(z)}{B(z)}\
&=lim_{zto infty}frac{2zleft(1+z+cdots+z^9right)left(1+2z+cdots+9z^8right)}{left(1+z+cdots+z^9right)^2}\
&=lim_{ztoinfty}frac{2zleft(1+2z+cdots+9z^8right)}{1+z+cdots+z^9}\
&,,color{blue}{=18}
end{align*}



The purpose is to analyse the coefficients
begin{align*}
[z^K]A(z)cdot B(z)^k
end{align*}

when $K$ and $k$ are linearly related. In order to do so the condition $K<Tk$ will be imposed which is inherent in the nature of the problem. Note that in our case we have $K=9k$ and with $T=18$ the condition $K<Tk$ evaluates to $9k<18k$ which is valid.



We also need a quantity $zeta$ which is introduced in Proposition VIII.7 and since this is a useful and easily derived upper bound for the coefficients we note




Proposition VIII.7 (Saddle-point bounds for large powers):




  • Consider functions $A(z)$ and $B(z)$ satisfying the conditions L1,L2,L3 above. Let $lambda$ be a positive number with $0<lambda <T$ and let $zeta$ be the unique positive root of the equation


begin{align*}
zetafrac{B^{prime}{(zeta)}}{B(zeta)}=lambda tag{4}
end{align*}



Then, for $K=lambda k$ an integer; one has
begin{align*}
[z^K]A(z)cdot B(z)^kleq A(zeta)B(zeta)^kzeta^{-K}tag{5}
end{align*}




We start with calculating the roots of (4). We set $lambda =9$ and obtain according to (4)
begin{align*}
zfrac{B^{prime}(z)}{B(z)}&=9\
end{align*}

which gives with $B(z)=left(1+z+cdots+z^9right)^2$:
begin{align*}
color{blue}{9z^9+7z^8+5z^7+3z^6+z^5-z^4-3z^3-5z^2-7z-9=0}
end{align*}

from which we easily derive the positive root $color{blue}{zeta =1}$.



We find as upper bound according to (5)
begin{align*}
[z^{9k}]left(frac{1-z^{10}}{1-z}right)^{2k}leq left(sum_{k=0}^91right)^{2k}cdot 1^{-9k}=10^{2k}
end{align*}

This upper bound isn't really sharp but it may be useful whenever only rough order of magnitude estimates are sought.



Now we are well prepared for the main theorem.




Theorem VIII.8 (Saddle-point estimates of large powers).





  • Under the conditions of Proposition VIII.7, with $lambda = K/k$, one has
    begin{align*}
    color{blue}{[z^K]A(z)cdot B(z)^k=A(zeta)frac{B(zeta)^k}{zeta^{K+1}sqrt{2pi n xi}}left(1+o(1)right)}tag{6}
    end{align*}



    where $zeta$ is the unique root of $zeta B^{prime}(zeta)B(zeta)=lambda$ and




begin{align*}
xi=frac{d^2}{dzeta^2}left(log B(zeta)-lambdalog zetaright).tag{7}
end{align*}




  • In addition, a full expansion in descending powers of $k$ exists.


These estimates hold uniformly for $lambda$ in any compact interval of $(0,T)$, i.e., any interval $[lambda^{prime},lambda^{primeprime}]$
with $0<lambda^{prime}<lambda^{primeprime}<T$, where $T$ is the spread.




Now it's time to harvest. At first we calculate $xi$ according to (7). We obtain with $B(z)=left(1+z+cdots+z^9right)^{2}$ and $lambda=9$:
begin{align*}
color{blue}{xi}&=left.left(frac{d^2}{dz^2}left(log (B(z)-9log zright)right)right|_{z=1}\
&=left.left(frac{d}{dz}left(frac{B^{prime}(z)}{B(z)}-frac{9}{z}right)right)right|_{z=1}\
&=frac{B^{primeprime}(1)}{B(1)}-left(frac{B^{prime}(1)}{B(1)}right)^2+9\
&=frac{177}{2}-81+9\
&,,color{blue}{=frac{33}{2}}tag{8}
end{align*}




Putting all together into (6) we finally obtain with $B(zeta)=B(1)=left.left(1+z+cdots+z^9right)^{2k}right|_{z=1}=10^{2k}$:
begin{align*}
color{blue}{[z^{9k}]left(frac{1-z^{10}}{1-z}right)^{2k}}
&=frac{10^{2k}}{1^{9k+1}sqrt{2pi k frac{33}{2}}}(1+o(1))\
&,,color{blue}{=10^{2k}frac{1}{sqrt{33pi k}}(1+o(1))}
end{align*}



in accordance with the claim (3).



Note: We have according to theorem VIII.8 the possibility to calculate a full expansion in descending powers of $k$. We can also study asymptotic expansions of $[z^K]B(z)^{k}$ for other quantities of $K$ as long as we fulfill the spread condition $K<Tk$.







share|cite|improve this answer











$endgroup$













  • $begingroup$
    Thank you, those are useful references!
    $endgroup$
    – ShreevatsaR
    Jan 12 at 16:56










  • $begingroup$
    @ShreevatsaR: You're welcome.
    $endgroup$
    – Markus Scheuer
    Jan 12 at 16:58










  • $begingroup$
    Thanks very much for a very clear and informative answer. I've often heard of the saddle-point method but never got around to learning it (e.g. didn't get that far in the Analytic Combinatorics book); now I have some idea. So if I understand this answer correctly, this gives the asymptotics of the largest coefficient... and we need to appeal to probability arguments etc to show that for the other coefficients, the probability drops off exponentially from this main (largest) term.
    $endgroup$
    – ShreevatsaR
    Jan 15 at 19:25












  • $begingroup$
    @ShreevatsaR: You're welcome and I agree with your considerations. I'd like to point to the introductory paragraph (the first one) in section VIII.9: Saddle-points and probability distributions. This section might go in the directions you are interested in.
    $endgroup$
    – Markus Scheuer
    Jan 15 at 19:36





















3












$begingroup$

You can prove that your asymptotic expression is correct using the Edgeworth series.



Let $F_k$ be the cdf for $sqrt{frac{k}{8.25}}(S_k/k-4.5)$. By the Central Limit theorem, $F_k(x)$ is approximately equal to $Phi(x)$. Specifically, the Edgeworth series shows that
$$
F_k(x)=Phi(x) -frac{lambda_3}{6sqrt k}Phi'''(x)+O(k^{-1})
$$

Here, $lambda_3$ is the skewness of a single random digit. Since this skewness is zero, as the distribution is symmetric about $[0,9]$, we get
$$
F_k(x)=Phi(x)+O(k^{-1}).
$$

This shows the error in the central limit approximation is linear in $k$. Therefore,
begin{align}
P(S_k=n)
&=P(n-tfrac12<S_kle n+tfrac12)
\&=F_kleft(frac{n+frac12-4.5k}{sqrt{8.25 k}}right)-F_kleft(frac{n-frac12-4.5k}{sqrt{8.25 k}}right)
\&=Phileft(frac{n+frac12-4.5k}{sqrt{8.25 k}}right)-Phileft(frac{n-frac12-4.5k}{sqrt{8.25 k}}right)+O(k^{-1})
\&stackrel1= Phi'(xi)cdotfrac1{sqrt{8.25k}}+O(k^{-1})
\&= frac1{sqrt{8.25k}}left(Phi'(xi)-Phi'left(frac{n-4.5k}{sqrt{8.25k}}right)+Phi'left(frac{n-4.5k}{sqrt{8.25k}}right)right)+O(k^{-1})
\&stackrel2= frac1{sqrt{8.25k}}Phi'left(frac{n-4.5k}{sqrt{8.25k}}right)+frac1{8.25k}Phi''(eta)+O(k^{-1})
\&stackrel3= frac1{sqrt{8.25k}}Phi'left(frac{n-4.5k}{sqrt{8.25k}}right)+O(k^{-1})
end{align}



Explanations:




  1. Here, we are applying the mean value theorem. $xi$ is a number between $frac{npmfrac12-4.5k}{sqrt{8.25 k}}$.


  2. Now we apply the mean value theorem to $Phi'$. Here, $eta$ is a number between $xi$ and $frac{n-4.5k}{sqrt{8.25 k}}$


  3. We can absorb the $frac1{8.25k}Phi''(eta)$ into the $O(k^{-1})$ because $Phi''$ is bounded.



This is exactly the asymptotic expansion you guessed; however, the above rigorously shows that your answer is correct, with the error decreasing linearly as $ktoinfty$.






share|cite|improve this answer









$endgroup$













  • $begingroup$
    Actually, this is omitting some details; in order for the Edgeworth series to work when the variables are integer valued, you need to add in a correction term which is periodic, as described in jstor.org/stable/2242145?seq=1#metadata_info_tab_contents. This term ends up canceling out; I will add in more details later.
    $endgroup$
    – Mike Earnest
    Jan 12 at 20:03










  • $begingroup$
    Wow these look like powerful techniques; thanks for sharing.
    $endgroup$
    – ShreevatsaR
    Jan 12 at 20:16










  • $begingroup$
    @ShreevatsaR I said I would post more info, but I have tried and been unable to figure out the correct result. I would suggest you try to look into the Edgeworth series, and specifically the series for lattice variables, and see if you can figure it out.
    $endgroup$
    – Mike Earnest
    Jan 14 at 23:40










  • $begingroup$
    Sure, thanks for your effort and for a very useful lead, and what matters is that I learned something new and useful... yes I'll try to look into this further too.
    $endgroup$
    – ShreevatsaR
    Jan 15 at 18:45



















2





+200







$begingroup$

Each digit in a $(r+1)$-ary base is a discrete uniform random variable, with support $[0,r]$.

The relevant mean and variance are
$$
mu = {r over 2}quad sigma ^{,2} = {{left( {r + 1} right)^{,2} - 1} over {12}}
$$



By the Central Limit Theorem the sum$n$ of $k$ of them will tend to be Normally distributed
with mean $k mu$ and variance $k sigma ^2$.

That is the expression that you give will tend (very quickly) to
$$
{1 over {sqrt {2pi ksigma ^{,2} } }}e^{, - ,{{left( {n - kmu } right)^{,2} } over {2ksigma ^{,2} }}}
= {{sqrt {6/pi } } over {sqrt {k,rleft( {r + 2} right)} }}e^{, - ,6{{left( {n - kr/2} right)^{,2} } over {k,rleft( {r + 2} right)}}}
$$



Then just replace $r=9$.



That for what concerns the asymptotic distribution.



Concerning your other questions, I did not understood properly what you want to compute.



------ Addendum in reply to your comment ------



1) A discrete uniform variable $0 le n le r$ is approximable to a continuous uniform
variable $-1/2 le nu le r+1/2$, with $p(n) approx p(n-1/2 le nu le n+1/2)$.



2) If $p(n;;,r,,k)$ denotes the probability above (either "exact" or "approximated") to get $n$ as the sum of $k$ $(r+1)$-ary digits (i.i.d.),
then its complement is
$$
q(n;;,r,,k) = 1 - p(n;;,r,,k)
$$

So, the probability that $n$ is not attained as the sum of either $1$, or $2$, .., or $m$ digits will be
$$
Q(n;;,r,,m) = prodlimits_{1, le ,k, le ,m} {q(n;;,r,,k)} = prodlimits_{1, le ,k, le ,m} {left( {1 - p(n;;,r,,k)} right)}
$$

For small $p(n;;,r,,k)$ (i.e. high $r,k$) we can approximate the above as
$$
eqalign{
& ln Q(n;;,r,,m) = sumlimits_{1, le ,k, le ,m} {ln left( {1 - p(n;;,r,,k)} right)} approx cr
& approx - sumlimits_{1, le ,k, le ,m} {p(n;;,r,,k) + Oleft( {p(n;;,r,,k)^{,2} } right)} cr}
$$



3) Finally note that the exact $p(n;;,r,,k)$ can be better written as
$$
eqalign{
& p(n;;,r,,k) = {{N_{,b} (n,r,k)} over {left( {r + 1} right)^{,m} }} = cr
& = sumlimits_{left( {0, le } right),,j,,left( { le ,{n over {r + 1}}, le ,k} right)} {left( { - 1} right)^j binom{k}{j}
binom{n + k - 1 - jleft( {r + 1} right)}{ n - jleft( {r + 1} right)}
} cr}
$$



which, as thoroughly explained in this related post, gives the advantage that the sum limits are implicit in the binomial coefficient,
thereby simplifying its algebraic manipulation.






share|cite|improve this answer











$endgroup$













  • $begingroup$
    Thanks… (1) can you explain how you deal with the fact that the normal distribution is a continuous one and we're asking about the sum being exactly one $n$ (a point)? (2) My second question was about summing the above over all integers $k$. In other words, first we take the sum $S_1$ of one random digit, then the sum $S_2$ of two random digits, then the sum $S_3$ of three random digits and so on, and ask what is the probability that $n$ is a member of this sequence. We have one normal distribution for each $k$, and we want the total probability distribution that falls on $n$.
    $endgroup$
    – ShreevatsaR
    Jan 7 at 3:57










  • $begingroup$
    @ShreevatsaR: I expanded my answer in reply to your comment: does it meet your requirements ?
    $endgroup$
    – G Cab
    Jan 7 at 17:21










  • $begingroup$
    Thank you very much (+1). I'll think about this a bit more but I think the initial part matches. BTW for the second part, I was interested in the case where (in your notation) $m to infty$ -- note that as $k$ becomes larger than than some threshold, again the probability of hitting $n$ starts falling off exponentially, so it should be possible to sum over all $k$ and get an expression $Q(n; r)$ (in your notation) that depends only on $n$ (and $r$). Maybe not even $n$...
    $endgroup$
    – ShreevatsaR
    Jan 7 at 18:36










  • $begingroup$
    increasing $k$ increases the mean of the gaussian $p(n,r,k)$, while the std dev. increases with $sqrt{k}$. So, keeping $n$ fixed, the gaussian will move towards and then beyond it, with an $O(exp(-k)/ sqrt{k})$ which is certainly summable.
    $endgroup$
    – G Cab
    Jan 7 at 19:22










  • $begingroup$
    Yes exactly; that's what I expect and that's why I'd like to know asymptotically what the sum is equal to. Maybe I'll post a separate question about how to sum it over all $k$. :-) (and accept this one after a while).
    $endgroup$
    – ShreevatsaR
    Jan 7 at 20:25



















1












$begingroup$

$newcommand{bbx}[1]{,bbox[15px,border:1px groove navy]{displaystyle{#1}},}
newcommand{braces}[1]{leftlbrace,{#1},rightrbrace}
newcommand{bracks}[1]{leftlbrack,{#1},rightrbrack}
newcommand{dd}{mathrm{d}}
newcommand{ds}[1]{displaystyle{#1}}
newcommand{expo}[1]{,mathrm{e}^{#1},}
newcommand{ic}{mathrm{i}}
newcommand{mc}[1]{mathcal{#1}}
newcommand{mrm}[1]{mathrm{#1}}
newcommand{pars}[1]{left(,{#1},right)}
newcommand{partiald}[3]{frac{partial^{#1} #2}{partial #3^{#1}}}
newcommand{root}[2]{,sqrt[#1]{,{#2},},}
newcommand{totald}[3]{frac{mathrm{d}^{#1} #2}{mathrm{d} #3^{#1}}}
newcommand{verts}[1]{leftvert,{#1},rightvert}$





  • Let $X_{1},ldots,X_{k}$ each be random digits. That is, they are independent random variables each uniformly distributed over the finite set $braces{0, 1, 2, 3, 4, 5, 6, 7, 8, 9}$.

  • Let $S equiv X_{1} + cdots + X_{k}$.

  • Given some integer $n$, what is the probability that $S = n$ ?.




begin{align}
&mathbb{P}bracks{X_{1} + cdots + X_{k} = n} equiv
bbox[10px,#ffd]{sum_{x_{1} = 0}^{9}{1 over 10}cdots
sum_{x_{k} = 0}^{9}{1 over 10}
bracks{z^{n}}z^{x_{1} + cdots + x_{k}}}
\[5mm] = &
{1 over 10^{k}}bracks{z^{n}}pars{sum_{x = 0}^{9}z^{x}}^{k} = {1 over 10^{k}}bracks{z^{n}}pars{z^{10} - 1 over z - 1}^{k}
\[5mm] = &
{1 over 10^{k}}bracks{z^{n}}
pars{1 - z^{10}}^{k}pars{1 - z}^{-k}
\[5mm] = &
{1 over 10^{k}}bracks{z^{n}}
bracks{sum_{ell = 0}^{k}{k choose ell}pars{-z^{10}}^{ell}}
bracks{sum_{m = 0}^{infty}{-k choose m}pars{-z}^{m}}
\[5mm] = &
{1 over 10^{k}}
sum_{ell = 0}^{k}{k choose ell}sum_{m = 0}^{infty}
bracks{{k + m - 1 choose m}pars{-1}^{m}}pars{-1}^{ell + m}
bracks{10ell + m = n}
\[5mm] = &
{1 over 10^{k}}
sum_{ell = 0}^{k}{k choose ell}pars{-1}^{ell}
sum_{m = 0}^{infty}{k + m - 1 choose k - 1}
bracks{m = n - 10ell}
\[5mm] = &
{1 over 10^{k}}
sum_{ell = 0}^{k}{k choose ell}pars{-1}^{ell}
{k + n - 10ell - 1 choose k - 1}bracks{n - 10ell geq 0}
\[5mm] = &
bbx{{1 over 10^{k}}
sum_{ell = 0}^{color{red}{M}}{k choose ell}
{k + n - 1 - 10ell choose k - 1}pars{-1}^{ell},,quad
color{red}{M} equiv
minbraces{k,leftlfloor{n over 10}rightrfloor}}
end{align}






share|cite|improve this answer











$endgroup$









  • 1




    $begingroup$
    This is already mentioned in the question (after the list of 19 other questions on this site). It's not useful because I want an asymptotic expression for large $n$ and $k$.
    $endgroup$
    – ShreevatsaR
    Jan 6 at 21:21










  • $begingroup$
    But thanks for taking the effort of typing this up, and please let me know how I could have written the question more clearly to avoid this — maybe assume that not everyone will read the whole question, and move this part to the top?
    $endgroup$
    – ShreevatsaR
    Jan 6 at 21:25










  • $begingroup$
    I've moved the formula from the middle to the top of the question — sorry for wasting your time; it didn't occur to me that someone might not read the whole question and end up writing something that's already mentioned in the question and in the answers to several other questions. Hope it's better now.
    $endgroup$
    – ShreevatsaR
    Jan 6 at 22:05










  • $begingroup$
    @ShreevatsaR Don't worry about it. It's fine you recognize my answer as useful one. Thanks.
    $endgroup$
    – Felix Marin
    Jan 7 at 17:41












Your Answer





StackExchange.ifUsing("editor", function () {
return StackExchange.using("mathjaxEditing", function () {
StackExchange.MarkdownEditor.creationCallbacks.add(function (editor, postfix) {
StackExchange.mathjaxEditing.prepareWmdForMathJax(editor, postfix, [["$", "$"], ["\\(","\\)"]]);
});
});
}, "mathjax-editing");

StackExchange.ready(function() {
var channelOptions = {
tags: "".split(" "),
id: "69"
};
initTagRenderer("".split(" "), "".split(" "), channelOptions);

StackExchange.using("externalEditor", function() {
// Have to fire editor after snippets, if snippets enabled
if (StackExchange.settings.snippets.snippetsEnabled) {
StackExchange.using("snippets", function() {
createEditor();
});
}
else {
createEditor();
}
});

function createEditor() {
StackExchange.prepareEditor({
heartbeatType: 'answer',
autoActivateHeartbeat: false,
convertImagesToLinks: true,
noModals: true,
showLowRepImageUploadWarning: true,
reputationToPostImages: 10,
bindNavPrevention: true,
postfix: "",
imageUploader: {
brandingHtml: "Powered by u003ca class="icon-imgur-white" href="https://imgur.com/"u003eu003c/au003e",
contentPolicyHtml: "User contributions licensed under u003ca href="https://creativecommons.org/licenses/by-sa/3.0/"u003ecc by-sa 3.0 with attribution requiredu003c/au003e u003ca href="https://stackoverflow.com/legal/content-policy"u003e(content policy)u003c/au003e",
allowUrls: true
},
noCode: true, onDemand: true,
discardSelector: ".discard-answer"
,immediatelyShowMarkdownHelp:true
});


}
});














draft saved

draft discarded


















StackExchange.ready(
function () {
StackExchange.openid.initPostLogin('.new-post-login', 'https%3a%2f%2fmath.stackexchange.com%2fquestions%2f3063646%2fwhat-is-the-probability-that-the-sum-of-digits-of-a-random-k-digit-number-is%23new-answer', 'question_page');
}
);

Post as a guest















Required, but never shown

























4 Answers
4






active

oldest

votes








4 Answers
4






active

oldest

votes









active

oldest

votes






active

oldest

votes









4












$begingroup$

Here we show the main term $frac{2}{sqrt{pi}}frac{1}{sqrt{66k}}$ in OPs asymptotic expansion is correct using the saddle-point method.




The coefficients in the expansion of the polynomials
begin{align*}
left(frac{1-z^{10}}{1-z}right)^{k}=left(1+z+z^2+z^3+z^4+z^5+z^6+z^7+z^8+z^9right)^{k}tag{1}
end{align*}

form a unimodal sequence. Taking even values $2k$ the maximum is the coefficient of the middle term
begin{align*}
[z^{9k}]left(frac{1-z^{10}}{1-z}right)^{2k}qquad kgeq 0tag{2}
end{align*}




The coefficients in (2) form a sequence starting with $(1,10,670, 55,252,4,816,030,ldots)$. These numbers are called Lucky ticket numbers and are stored in OEIS. In fact they are the even central decanomial coefficients which are the largest coefficients in the expansion of (1).




They also cite an asymptotic formula for the largest coefficient of $left(1+z+cdots+z^qright)^k$ which gives in the case (2)
begin{align*}
color{blue}{[z^{9k}]left(frac{1-z^{10}}{1-z}right)^{2k}sim 10^{2k}frac{1}{sqrt{33pi k}}}tag{3}
end{align*}

corresponding to OPs main term in his expansion ($k$ even).




We can prove the asymptotic expansion (3) using the saddle-point method. In order to do so we closely follow section VIII.8 Large Powers in Analytic Combinatorics by P. Flajolet and R. Sedgewick. We also give a little bit of surrounding information to ease readability.




VIII.8 Large powers:





  • (p. 585): The extraction of coefficients in powers of a fixed function and more generally in functions of the form $A(z)B(z)^k$ constitutes a prototypical and easy application of the saddle-point method. We will accordingly be concerned here with the problem of estimating
    begin{align*}
    [z^K]A(z)cdot B(z)^k=frac{1}{2pi i}oint A(z)B(z)^kfrac{dz}{z^{K+1}}
    end{align*}

    as both $k$ and $K$ get large.




In our situation (3) we have $A(z)=1$ and $B(z)=left(1+z+z^2+cdots+z^9right)^2$ with $K=9k$. What follows are conditions which need to be fulfilled by $A(z)$ and $B(z)$ in order to apply the saddle-point method.




VIII. 8.1. Large powers: saddle-point bounds: We consider throughout this section two fixed functions, $A(z)$ and $B(z)$ satisfying the following conditions.




  • L1: The functions $A(z)=sum_{jgeq 0}a_jz^j$ and $B(z)=sum_{jgeq 0}a_jz^j$ are analytic at $0$ and have non-negative coefficients; furthermore it is assumed (without loss of generality) that $B(0) ne 0$.


  • L2: The function $B(z)$ is aperiodic in the sense that $gcd{j|b_j>0}=1$. (Thus $B(z)$ is not a function of the form $beta(z^p)$ for some integer $pgeq 2$ and some $beta$ analytic at $0$.)


  • L3: Let $Rleq infty$ be the radius of convergence of $B(z)$; the radius of convergence of $A(z)$ is at least as large as $R$.





We observe conditions L1 to L3 are fulfilled for $A(z)=1$ and $B(z)=left(1+z+z^2+cdots+z^9right)^2$ with radius of convergence $R=infty$. In the following we need the quantity $T$ called spread which is defined as



begin{align*}
color{blue}{T}&:=lim_{zto R^{-}}frac{zB^{prime}(z)}{B(z)}\
&=lim_{zto infty}frac{2zleft(1+z+cdots+z^9right)left(1+2z+cdots+9z^8right)}{left(1+z+cdots+z^9right)^2}\
&=lim_{ztoinfty}frac{2zleft(1+2z+cdots+9z^8right)}{1+z+cdots+z^9}\
&,,color{blue}{=18}
end{align*}



The purpose is to analyse the coefficients
begin{align*}
[z^K]A(z)cdot B(z)^k
end{align*}

when $K$ and $k$ are linearly related. In order to do so the condition $K<Tk$ will be imposed which is inherent in the nature of the problem. Note that in our case we have $K=9k$ and with $T=18$ the condition $K<Tk$ evaluates to $9k<18k$ which is valid.



We also need a quantity $zeta$ which is introduced in Proposition VIII.7 and since this is a useful and easily derived upper bound for the coefficients we note




Proposition VIII.7 (Saddle-point bounds for large powers):




  • Consider functions $A(z)$ and $B(z)$ satisfying the conditions L1,L2,L3 above. Let $lambda$ be a positive number with $0<lambda <T$ and let $zeta$ be the unique positive root of the equation


begin{align*}
zetafrac{B^{prime}{(zeta)}}{B(zeta)}=lambda tag{4}
end{align*}



Then, for $K=lambda k$ an integer; one has
begin{align*}
[z^K]A(z)cdot B(z)^kleq A(zeta)B(zeta)^kzeta^{-K}tag{5}
end{align*}




We start with calculating the roots of (4). We set $lambda =9$ and obtain according to (4)
begin{align*}
zfrac{B^{prime}(z)}{B(z)}&=9\
end{align*}

which gives with $B(z)=left(1+z+cdots+z^9right)^2$:
begin{align*}
color{blue}{9z^9+7z^8+5z^7+3z^6+z^5-z^4-3z^3-5z^2-7z-9=0}
end{align*}

from which we easily derive the positive root $color{blue}{zeta =1}$.



We find as upper bound according to (5)
begin{align*}
[z^{9k}]left(frac{1-z^{10}}{1-z}right)^{2k}leq left(sum_{k=0}^91right)^{2k}cdot 1^{-9k}=10^{2k}
end{align*}

This upper bound isn't really sharp but it may be useful whenever only rough order of magnitude estimates are sought.



Now we are well prepared for the main theorem.




Theorem VIII.8 (Saddle-point estimates of large powers).





  • Under the conditions of Proposition VIII.7, with $lambda = K/k$, one has
    begin{align*}
    color{blue}{[z^K]A(z)cdot B(z)^k=A(zeta)frac{B(zeta)^k}{zeta^{K+1}sqrt{2pi n xi}}left(1+o(1)right)}tag{6}
    end{align*}



    where $zeta$ is the unique root of $zeta B^{prime}(zeta)B(zeta)=lambda$ and




begin{align*}
xi=frac{d^2}{dzeta^2}left(log B(zeta)-lambdalog zetaright).tag{7}
end{align*}




  • In addition, a full expansion in descending powers of $k$ exists.


These estimates hold uniformly for $lambda$ in any compact interval of $(0,T)$, i.e., any interval $[lambda^{prime},lambda^{primeprime}]$
with $0<lambda^{prime}<lambda^{primeprime}<T$, where $T$ is the spread.




Now it's time to harvest. At first we calculate $xi$ according to (7). We obtain with $B(z)=left(1+z+cdots+z^9right)^{2}$ and $lambda=9$:
begin{align*}
color{blue}{xi}&=left.left(frac{d^2}{dz^2}left(log (B(z)-9log zright)right)right|_{z=1}\
&=left.left(frac{d}{dz}left(frac{B^{prime}(z)}{B(z)}-frac{9}{z}right)right)right|_{z=1}\
&=frac{B^{primeprime}(1)}{B(1)}-left(frac{B^{prime}(1)}{B(1)}right)^2+9\
&=frac{177}{2}-81+9\
&,,color{blue}{=frac{33}{2}}tag{8}
end{align*}




Putting all together into (6) we finally obtain with $B(zeta)=B(1)=left.left(1+z+cdots+z^9right)^{2k}right|_{z=1}=10^{2k}$:
begin{align*}
color{blue}{[z^{9k}]left(frac{1-z^{10}}{1-z}right)^{2k}}
&=frac{10^{2k}}{1^{9k+1}sqrt{2pi k frac{33}{2}}}(1+o(1))\
&,,color{blue}{=10^{2k}frac{1}{sqrt{33pi k}}(1+o(1))}
end{align*}



in accordance with the claim (3).



Note: We have according to theorem VIII.8 the possibility to calculate a full expansion in descending powers of $k$. We can also study asymptotic expansions of $[z^K]B(z)^{k}$ for other quantities of $K$ as long as we fulfill the spread condition $K<Tk$.







share|cite|improve this answer











$endgroup$













  • $begingroup$
    Thank you, those are useful references!
    $endgroup$
    – ShreevatsaR
    Jan 12 at 16:56










  • $begingroup$
    @ShreevatsaR: You're welcome.
    $endgroup$
    – Markus Scheuer
    Jan 12 at 16:58










  • $begingroup$
    Thanks very much for a very clear and informative answer. I've often heard of the saddle-point method but never got around to learning it (e.g. didn't get that far in the Analytic Combinatorics book); now I have some idea. So if I understand this answer correctly, this gives the asymptotics of the largest coefficient... and we need to appeal to probability arguments etc to show that for the other coefficients, the probability drops off exponentially from this main (largest) term.
    $endgroup$
    – ShreevatsaR
    Jan 15 at 19:25












  • $begingroup$
    @ShreevatsaR: You're welcome and I agree with your considerations. I'd like to point to the introductory paragraph (the first one) in section VIII.9: Saddle-points and probability distributions. This section might go in the directions you are interested in.
    $endgroup$
    – Markus Scheuer
    Jan 15 at 19:36


















4












$begingroup$

Here we show the main term $frac{2}{sqrt{pi}}frac{1}{sqrt{66k}}$ in OPs asymptotic expansion is correct using the saddle-point method.




The coefficients in the expansion of the polynomials
begin{align*}
left(frac{1-z^{10}}{1-z}right)^{k}=left(1+z+z^2+z^3+z^4+z^5+z^6+z^7+z^8+z^9right)^{k}tag{1}
end{align*}

form a unimodal sequence. Taking even values $2k$ the maximum is the coefficient of the middle term
begin{align*}
[z^{9k}]left(frac{1-z^{10}}{1-z}right)^{2k}qquad kgeq 0tag{2}
end{align*}




The coefficients in (2) form a sequence starting with $(1,10,670, 55,252,4,816,030,ldots)$. These numbers are called Lucky ticket numbers and are stored in OEIS. In fact they are the even central decanomial coefficients which are the largest coefficients in the expansion of (1).




They also cite an asymptotic formula for the largest coefficient of $left(1+z+cdots+z^qright)^k$ which gives in the case (2)
begin{align*}
color{blue}{[z^{9k}]left(frac{1-z^{10}}{1-z}right)^{2k}sim 10^{2k}frac{1}{sqrt{33pi k}}}tag{3}
end{align*}

corresponding to OPs main term in his expansion ($k$ even).




We can prove the asymptotic expansion (3) using the saddle-point method. In order to do so we closely follow section VIII.8 Large Powers in Analytic Combinatorics by P. Flajolet and R. Sedgewick. We also give a little bit of surrounding information to ease readability.




VIII.8 Large powers:





  • (p. 585): The extraction of coefficients in powers of a fixed function and more generally in functions of the form $A(z)B(z)^k$ constitutes a prototypical and easy application of the saddle-point method. We will accordingly be concerned here with the problem of estimating
    begin{align*}
    [z^K]A(z)cdot B(z)^k=frac{1}{2pi i}oint A(z)B(z)^kfrac{dz}{z^{K+1}}
    end{align*}

    as both $k$ and $K$ get large.




In our situation (3) we have $A(z)=1$ and $B(z)=left(1+z+z^2+cdots+z^9right)^2$ with $K=9k$. What follows are conditions which need to be fulfilled by $A(z)$ and $B(z)$ in order to apply the saddle-point method.




VIII. 8.1. Large powers: saddle-point bounds: We consider throughout this section two fixed functions, $A(z)$ and $B(z)$ satisfying the following conditions.




  • L1: The functions $A(z)=sum_{jgeq 0}a_jz^j$ and $B(z)=sum_{jgeq 0}a_jz^j$ are analytic at $0$ and have non-negative coefficients; furthermore it is assumed (without loss of generality) that $B(0) ne 0$.


  • L2: The function $B(z)$ is aperiodic in the sense that $gcd{j|b_j>0}=1$. (Thus $B(z)$ is not a function of the form $beta(z^p)$ for some integer $pgeq 2$ and some $beta$ analytic at $0$.)


  • L3: Let $Rleq infty$ be the radius of convergence of $B(z)$; the radius of convergence of $A(z)$ is at least as large as $R$.





We observe conditions L1 to L3 are fulfilled for $A(z)=1$ and $B(z)=left(1+z+z^2+cdots+z^9right)^2$ with radius of convergence $R=infty$. In the following we need the quantity $T$ called spread which is defined as



begin{align*}
color{blue}{T}&:=lim_{zto R^{-}}frac{zB^{prime}(z)}{B(z)}\
&=lim_{zto infty}frac{2zleft(1+z+cdots+z^9right)left(1+2z+cdots+9z^8right)}{left(1+z+cdots+z^9right)^2}\
&=lim_{ztoinfty}frac{2zleft(1+2z+cdots+9z^8right)}{1+z+cdots+z^9}\
&,,color{blue}{=18}
end{align*}



The purpose is to analyse the coefficients
begin{align*}
[z^K]A(z)cdot B(z)^k
end{align*}

when $K$ and $k$ are linearly related. In order to do so the condition $K<Tk$ will be imposed which is inherent in the nature of the problem. Note that in our case we have $K=9k$ and with $T=18$ the condition $K<Tk$ evaluates to $9k<18k$ which is valid.



We also need a quantity $zeta$ which is introduced in Proposition VIII.7 and since this is a useful and easily derived upper bound for the coefficients we note




Proposition VIII.7 (Saddle-point bounds for large powers):




  • Consider functions $A(z)$ and $B(z)$ satisfying the conditions L1,L2,L3 above. Let $lambda$ be a positive number with $0<lambda <T$ and let $zeta$ be the unique positive root of the equation


begin{align*}
zetafrac{B^{prime}{(zeta)}}{B(zeta)}=lambda tag{4}
end{align*}



Then, for $K=lambda k$ an integer; one has
begin{align*}
[z^K]A(z)cdot B(z)^kleq A(zeta)B(zeta)^kzeta^{-K}tag{5}
end{align*}




We start with calculating the roots of (4). We set $lambda =9$ and obtain according to (4)
begin{align*}
zfrac{B^{prime}(z)}{B(z)}&=9\
end{align*}

which gives with $B(z)=left(1+z+cdots+z^9right)^2$:
begin{align*}
color{blue}{9z^9+7z^8+5z^7+3z^6+z^5-z^4-3z^3-5z^2-7z-9=0}
end{align*}

from which we easily derive the positive root $color{blue}{zeta =1}$.



We find as upper bound according to (5)
begin{align*}
[z^{9k}]left(frac{1-z^{10}}{1-z}right)^{2k}leq left(sum_{k=0}^91right)^{2k}cdot 1^{-9k}=10^{2k}
end{align*}

This upper bound isn't really sharp but it may be useful whenever only rough order of magnitude estimates are sought.



Now we are well prepared for the main theorem.




Theorem VIII.8 (Saddle-point estimates of large powers).





  • Under the conditions of Proposition VIII.7, with $lambda = K/k$, one has
    begin{align*}
    color{blue}{[z^K]A(z)cdot B(z)^k=A(zeta)frac{B(zeta)^k}{zeta^{K+1}sqrt{2pi n xi}}left(1+o(1)right)}tag{6}
    end{align*}



    where $zeta$ is the unique root of $zeta B^{prime}(zeta)B(zeta)=lambda$ and




begin{align*}
xi=frac{d^2}{dzeta^2}left(log B(zeta)-lambdalog zetaright).tag{7}
end{align*}




  • In addition, a full expansion in descending powers of $k$ exists.


These estimates hold uniformly for $lambda$ in any compact interval of $(0,T)$, i.e., any interval $[lambda^{prime},lambda^{primeprime}]$
with $0<lambda^{prime}<lambda^{primeprime}<T$, where $T$ is the spread.




Now it's time to harvest. At first we calculate $xi$ according to (7). We obtain with $B(z)=left(1+z+cdots+z^9right)^{2}$ and $lambda=9$:
begin{align*}
color{blue}{xi}&=left.left(frac{d^2}{dz^2}left(log (B(z)-9log zright)right)right|_{z=1}\
&=left.left(frac{d}{dz}left(frac{B^{prime}(z)}{B(z)}-frac{9}{z}right)right)right|_{z=1}\
&=frac{B^{primeprime}(1)}{B(1)}-left(frac{B^{prime}(1)}{B(1)}right)^2+9\
&=frac{177}{2}-81+9\
&,,color{blue}{=frac{33}{2}}tag{8}
end{align*}




Putting all together into (6) we finally obtain with $B(zeta)=B(1)=left.left(1+z+cdots+z^9right)^{2k}right|_{z=1}=10^{2k}$:
begin{align*}
color{blue}{[z^{9k}]left(frac{1-z^{10}}{1-z}right)^{2k}}
&=frac{10^{2k}}{1^{9k+1}sqrt{2pi k frac{33}{2}}}(1+o(1))\
&,,color{blue}{=10^{2k}frac{1}{sqrt{33pi k}}(1+o(1))}
end{align*}



in accordance with the claim (3).



Note: We have according to theorem VIII.8 the possibility to calculate a full expansion in descending powers of $k$. We can also study asymptotic expansions of $[z^K]B(z)^{k}$ for other quantities of $K$ as long as we fulfill the spread condition $K<Tk$.







share|cite|improve this answer











$endgroup$













  • $begingroup$
    Thank you, those are useful references!
    $endgroup$
    – ShreevatsaR
    Jan 12 at 16:56










  • $begingroup$
    @ShreevatsaR: You're welcome.
    $endgroup$
    – Markus Scheuer
    Jan 12 at 16:58










  • $begingroup$
    Thanks very much for a very clear and informative answer. I've often heard of the saddle-point method but never got around to learning it (e.g. didn't get that far in the Analytic Combinatorics book); now I have some idea. So if I understand this answer correctly, this gives the asymptotics of the largest coefficient... and we need to appeal to probability arguments etc to show that for the other coefficients, the probability drops off exponentially from this main (largest) term.
    $endgroup$
    – ShreevatsaR
    Jan 15 at 19:25












  • $begingroup$
    @ShreevatsaR: You're welcome and I agree with your considerations. I'd like to point to the introductory paragraph (the first one) in section VIII.9: Saddle-points and probability distributions. This section might go in the directions you are interested in.
    $endgroup$
    – Markus Scheuer
    Jan 15 at 19:36
















4












4








4





$begingroup$

Here we show the main term $frac{2}{sqrt{pi}}frac{1}{sqrt{66k}}$ in OPs asymptotic expansion is correct using the saddle-point method.




The coefficients in the expansion of the polynomials
begin{align*}
left(frac{1-z^{10}}{1-z}right)^{k}=left(1+z+z^2+z^3+z^4+z^5+z^6+z^7+z^8+z^9right)^{k}tag{1}
end{align*}

form a unimodal sequence. Taking even values $2k$ the maximum is the coefficient of the middle term
begin{align*}
[z^{9k}]left(frac{1-z^{10}}{1-z}right)^{2k}qquad kgeq 0tag{2}
end{align*}




The coefficients in (2) form a sequence starting with $(1,10,670, 55,252,4,816,030,ldots)$. These numbers are called Lucky ticket numbers and are stored in OEIS. In fact they are the even central decanomial coefficients which are the largest coefficients in the expansion of (1).




They also cite an asymptotic formula for the largest coefficient of $left(1+z+cdots+z^qright)^k$ which gives in the case (2)
begin{align*}
color{blue}{[z^{9k}]left(frac{1-z^{10}}{1-z}right)^{2k}sim 10^{2k}frac{1}{sqrt{33pi k}}}tag{3}
end{align*}

corresponding to OPs main term in his expansion ($k$ even).




We can prove the asymptotic expansion (3) using the saddle-point method. In order to do so we closely follow section VIII.8 Large Powers in Analytic Combinatorics by P. Flajolet and R. Sedgewick. We also give a little bit of surrounding information to ease readability.




VIII.8 Large powers:





  • (p. 585): The extraction of coefficients in powers of a fixed function and more generally in functions of the form $A(z)B(z)^k$ constitutes a prototypical and easy application of the saddle-point method. We will accordingly be concerned here with the problem of estimating
    begin{align*}
    [z^K]A(z)cdot B(z)^k=frac{1}{2pi i}oint A(z)B(z)^kfrac{dz}{z^{K+1}}
    end{align*}

    as both $k$ and $K$ get large.




In our situation (3) we have $A(z)=1$ and $B(z)=left(1+z+z^2+cdots+z^9right)^2$ with $K=9k$. What follows are conditions which need to be fulfilled by $A(z)$ and $B(z)$ in order to apply the saddle-point method.




VIII. 8.1. Large powers: saddle-point bounds: We consider throughout this section two fixed functions, $A(z)$ and $B(z)$ satisfying the following conditions.




  • L1: The functions $A(z)=sum_{jgeq 0}a_jz^j$ and $B(z)=sum_{jgeq 0}a_jz^j$ are analytic at $0$ and have non-negative coefficients; furthermore it is assumed (without loss of generality) that $B(0) ne 0$.


  • L2: The function $B(z)$ is aperiodic in the sense that $gcd{j|b_j>0}=1$. (Thus $B(z)$ is not a function of the form $beta(z^p)$ for some integer $pgeq 2$ and some $beta$ analytic at $0$.)


  • L3: Let $Rleq infty$ be the radius of convergence of $B(z)$; the radius of convergence of $A(z)$ is at least as large as $R$.





We observe conditions L1 to L3 are fulfilled for $A(z)=1$ and $B(z)=left(1+z+z^2+cdots+z^9right)^2$ with radius of convergence $R=infty$. In the following we need the quantity $T$ called spread which is defined as



begin{align*}
color{blue}{T}&:=lim_{zto R^{-}}frac{zB^{prime}(z)}{B(z)}\
&=lim_{zto infty}frac{2zleft(1+z+cdots+z^9right)left(1+2z+cdots+9z^8right)}{left(1+z+cdots+z^9right)^2}\
&=lim_{ztoinfty}frac{2zleft(1+2z+cdots+9z^8right)}{1+z+cdots+z^9}\
&,,color{blue}{=18}
end{align*}



The purpose is to analyse the coefficients
begin{align*}
[z^K]A(z)cdot B(z)^k
end{align*}

when $K$ and $k$ are linearly related. In order to do so the condition $K<Tk$ will be imposed which is inherent in the nature of the problem. Note that in our case we have $K=9k$ and with $T=18$ the condition $K<Tk$ evaluates to $9k<18k$ which is valid.



We also need a quantity $zeta$ which is introduced in Proposition VIII.7 and since this is a useful and easily derived upper bound for the coefficients we note




Proposition VIII.7 (Saddle-point bounds for large powers):




  • Consider functions $A(z)$ and $B(z)$ satisfying the conditions L1,L2,L3 above. Let $lambda$ be a positive number with $0<lambda <T$ and let $zeta$ be the unique positive root of the equation


begin{align*}
zetafrac{B^{prime}{(zeta)}}{B(zeta)}=lambda tag{4}
end{align*}



Then, for $K=lambda k$ an integer; one has
begin{align*}
[z^K]A(z)cdot B(z)^kleq A(zeta)B(zeta)^kzeta^{-K}tag{5}
end{align*}




We start with calculating the roots of (4). We set $lambda =9$ and obtain according to (4)
begin{align*}
zfrac{B^{prime}(z)}{B(z)}&=9\
end{align*}

which gives with $B(z)=left(1+z+cdots+z^9right)^2$:
begin{align*}
color{blue}{9z^9+7z^8+5z^7+3z^6+z^5-z^4-3z^3-5z^2-7z-9=0}
end{align*}

from which we easily derive the positive root $color{blue}{zeta =1}$.



We find as upper bound according to (5)
begin{align*}
[z^{9k}]left(frac{1-z^{10}}{1-z}right)^{2k}leq left(sum_{k=0}^91right)^{2k}cdot 1^{-9k}=10^{2k}
end{align*}

This upper bound isn't really sharp but it may be useful whenever only rough order of magnitude estimates are sought.



Now we are well prepared for the main theorem.




Theorem VIII.8 (Saddle-point estimates of large powers).





  • Under the conditions of Proposition VIII.7, with $lambda = K/k$, one has
    begin{align*}
    color{blue}{[z^K]A(z)cdot B(z)^k=A(zeta)frac{B(zeta)^k}{zeta^{K+1}sqrt{2pi n xi}}left(1+o(1)right)}tag{6}
    end{align*}



    where $zeta$ is the unique root of $zeta B^{prime}(zeta)B(zeta)=lambda$ and




begin{align*}
xi=frac{d^2}{dzeta^2}left(log B(zeta)-lambdalog zetaright).tag{7}
end{align*}




  • In addition, a full expansion in descending powers of $k$ exists.


These estimates hold uniformly for $lambda$ in any compact interval of $(0,T)$, i.e., any interval $[lambda^{prime},lambda^{primeprime}]$
with $0<lambda^{prime}<lambda^{primeprime}<T$, where $T$ is the spread.




Now it's time to harvest. At first we calculate $xi$ according to (7). We obtain with $B(z)=left(1+z+cdots+z^9right)^{2}$ and $lambda=9$:
begin{align*}
color{blue}{xi}&=left.left(frac{d^2}{dz^2}left(log (B(z)-9log zright)right)right|_{z=1}\
&=left.left(frac{d}{dz}left(frac{B^{prime}(z)}{B(z)}-frac{9}{z}right)right)right|_{z=1}\
&=frac{B^{primeprime}(1)}{B(1)}-left(frac{B^{prime}(1)}{B(1)}right)^2+9\
&=frac{177}{2}-81+9\
&,,color{blue}{=frac{33}{2}}tag{8}
end{align*}




Putting all together into (6) we finally obtain with $B(zeta)=B(1)=left.left(1+z+cdots+z^9right)^{2k}right|_{z=1}=10^{2k}$:
begin{align*}
color{blue}{[z^{9k}]left(frac{1-z^{10}}{1-z}right)^{2k}}
&=frac{10^{2k}}{1^{9k+1}sqrt{2pi k frac{33}{2}}}(1+o(1))\
&,,color{blue}{=10^{2k}frac{1}{sqrt{33pi k}}(1+o(1))}
end{align*}



in accordance with the claim (3).



Note: We have according to theorem VIII.8 the possibility to calculate a full expansion in descending powers of $k$. We can also study asymptotic expansions of $[z^K]B(z)^{k}$ for other quantities of $K$ as long as we fulfill the spread condition $K<Tk$.







share|cite|improve this answer











$endgroup$



Here we show the main term $frac{2}{sqrt{pi}}frac{1}{sqrt{66k}}$ in OPs asymptotic expansion is correct using the saddle-point method.




The coefficients in the expansion of the polynomials
begin{align*}
left(frac{1-z^{10}}{1-z}right)^{k}=left(1+z+z^2+z^3+z^4+z^5+z^6+z^7+z^8+z^9right)^{k}tag{1}
end{align*}

form a unimodal sequence. Taking even values $2k$ the maximum is the coefficient of the middle term
begin{align*}
[z^{9k}]left(frac{1-z^{10}}{1-z}right)^{2k}qquad kgeq 0tag{2}
end{align*}




The coefficients in (2) form a sequence starting with $(1,10,670, 55,252,4,816,030,ldots)$. These numbers are called Lucky ticket numbers and are stored in OEIS. In fact they are the even central decanomial coefficients which are the largest coefficients in the expansion of (1).




They also cite an asymptotic formula for the largest coefficient of $left(1+z+cdots+z^qright)^k$ which gives in the case (2)
begin{align*}
color{blue}{[z^{9k}]left(frac{1-z^{10}}{1-z}right)^{2k}sim 10^{2k}frac{1}{sqrt{33pi k}}}tag{3}
end{align*}

corresponding to OPs main term in his expansion ($k$ even).




We can prove the asymptotic expansion (3) using the saddle-point method. In order to do so we closely follow section VIII.8 Large Powers in Analytic Combinatorics by P. Flajolet and R. Sedgewick. We also give a little bit of surrounding information to ease readability.




VIII.8 Large powers:





  • (p. 585): The extraction of coefficients in powers of a fixed function and more generally in functions of the form $A(z)B(z)^k$ constitutes a prototypical and easy application of the saddle-point method. We will accordingly be concerned here with the problem of estimating
    begin{align*}
    [z^K]A(z)cdot B(z)^k=frac{1}{2pi i}oint A(z)B(z)^kfrac{dz}{z^{K+1}}
    end{align*}

    as both $k$ and $K$ get large.




In our situation (3) we have $A(z)=1$ and $B(z)=left(1+z+z^2+cdots+z^9right)^2$ with $K=9k$. What follows are conditions which need to be fulfilled by $A(z)$ and $B(z)$ in order to apply the saddle-point method.




VIII. 8.1. Large powers: saddle-point bounds: We consider throughout this section two fixed functions, $A(z)$ and $B(z)$ satisfying the following conditions.




  • L1: The functions $A(z)=sum_{jgeq 0}a_jz^j$ and $B(z)=sum_{jgeq 0}a_jz^j$ are analytic at $0$ and have non-negative coefficients; furthermore it is assumed (without loss of generality) that $B(0) ne 0$.


  • L2: The function $B(z)$ is aperiodic in the sense that $gcd{j|b_j>0}=1$. (Thus $B(z)$ is not a function of the form $beta(z^p)$ for some integer $pgeq 2$ and some $beta$ analytic at $0$.)


  • L3: Let $Rleq infty$ be the radius of convergence of $B(z)$; the radius of convergence of $A(z)$ is at least as large as $R$.





We observe conditions L1 to L3 are fulfilled for $A(z)=1$ and $B(z)=left(1+z+z^2+cdots+z^9right)^2$ with radius of convergence $R=infty$. In the following we need the quantity $T$ called spread which is defined as



begin{align*}
color{blue}{T}&:=lim_{zto R^{-}}frac{zB^{prime}(z)}{B(z)}\
&=lim_{zto infty}frac{2zleft(1+z+cdots+z^9right)left(1+2z+cdots+9z^8right)}{left(1+z+cdots+z^9right)^2}\
&=lim_{ztoinfty}frac{2zleft(1+2z+cdots+9z^8right)}{1+z+cdots+z^9}\
&,,color{blue}{=18}
end{align*}



The purpose is to analyse the coefficients
begin{align*}
[z^K]A(z)cdot B(z)^k
end{align*}

when $K$ and $k$ are linearly related. In order to do so the condition $K<Tk$ will be imposed which is inherent in the nature of the problem. Note that in our case we have $K=9k$ and with $T=18$ the condition $K<Tk$ evaluates to $9k<18k$ which is valid.



We also need a quantity $zeta$ which is introduced in Proposition VIII.7 and since this is a useful and easily derived upper bound for the coefficients we note




Proposition VIII.7 (Saddle-point bounds for large powers):




  • Consider functions $A(z)$ and $B(z)$ satisfying the conditions L1,L2,L3 above. Let $lambda$ be a positive number with $0<lambda <T$ and let $zeta$ be the unique positive root of the equation


begin{align*}
zetafrac{B^{prime}{(zeta)}}{B(zeta)}=lambda tag{4}
end{align*}



Then, for $K=lambda k$ an integer; one has
begin{align*}
[z^K]A(z)cdot B(z)^kleq A(zeta)B(zeta)^kzeta^{-K}tag{5}
end{align*}




We start with calculating the roots of (4). We set $lambda =9$ and obtain according to (4)
begin{align*}
zfrac{B^{prime}(z)}{B(z)}&=9\
end{align*}

which gives with $B(z)=left(1+z+cdots+z^9right)^2$:
begin{align*}
color{blue}{9z^9+7z^8+5z^7+3z^6+z^5-z^4-3z^3-5z^2-7z-9=0}
end{align*}

from which we easily derive the positive root $color{blue}{zeta =1}$.



We find as upper bound according to (5)
begin{align*}
[z^{9k}]left(frac{1-z^{10}}{1-z}right)^{2k}leq left(sum_{k=0}^91right)^{2k}cdot 1^{-9k}=10^{2k}
end{align*}

This upper bound isn't really sharp but it may be useful whenever only rough order of magnitude estimates are sought.



Now we are well prepared for the main theorem.




Theorem VIII.8 (Saddle-point estimates of large powers).





  • Under the conditions of Proposition VIII.7, with $lambda = K/k$, one has
    begin{align*}
    color{blue}{[z^K]A(z)cdot B(z)^k=A(zeta)frac{B(zeta)^k}{zeta^{K+1}sqrt{2pi n xi}}left(1+o(1)right)}tag{6}
    end{align*}



    where $zeta$ is the unique root of $zeta B^{prime}(zeta)B(zeta)=lambda$ and




begin{align*}
xi=frac{d^2}{dzeta^2}left(log B(zeta)-lambdalog zetaright).tag{7}
end{align*}




  • In addition, a full expansion in descending powers of $k$ exists.


These estimates hold uniformly for $lambda$ in any compact interval of $(0,T)$, i.e., any interval $[lambda^{prime},lambda^{primeprime}]$
with $0<lambda^{prime}<lambda^{primeprime}<T$, where $T$ is the spread.




Now it's time to harvest. At first we calculate $xi$ according to (7). We obtain with $B(z)=left(1+z+cdots+z^9right)^{2}$ and $lambda=9$:
begin{align*}
color{blue}{xi}&=left.left(frac{d^2}{dz^2}left(log (B(z)-9log zright)right)right|_{z=1}\
&=left.left(frac{d}{dz}left(frac{B^{prime}(z)}{B(z)}-frac{9}{z}right)right)right|_{z=1}\
&=frac{B^{primeprime}(1)}{B(1)}-left(frac{B^{prime}(1)}{B(1)}right)^2+9\
&=frac{177}{2}-81+9\
&,,color{blue}{=frac{33}{2}}tag{8}
end{align*}




Putting all together into (6) we finally obtain with $B(zeta)=B(1)=left.left(1+z+cdots+z^9right)^{2k}right|_{z=1}=10^{2k}$:
begin{align*}
color{blue}{[z^{9k}]left(frac{1-z^{10}}{1-z}right)^{2k}}
&=frac{10^{2k}}{1^{9k+1}sqrt{2pi k frac{33}{2}}}(1+o(1))\
&,,color{blue}{=10^{2k}frac{1}{sqrt{33pi k}}(1+o(1))}
end{align*}



in accordance with the claim (3).



Note: We have according to theorem VIII.8 the possibility to calculate a full expansion in descending powers of $k$. We can also study asymptotic expansions of $[z^K]B(z)^{k}$ for other quantities of $K$ as long as we fulfill the spread condition $K<Tk$.








share|cite|improve this answer














share|cite|improve this answer



share|cite|improve this answer








edited Jan 14 at 9:04

























answered Jan 12 at 16:46









Markus ScheuerMarkus Scheuer

64k460152




64k460152












  • $begingroup$
    Thank you, those are useful references!
    $endgroup$
    – ShreevatsaR
    Jan 12 at 16:56










  • $begingroup$
    @ShreevatsaR: You're welcome.
    $endgroup$
    – Markus Scheuer
    Jan 12 at 16:58










  • $begingroup$
    Thanks very much for a very clear and informative answer. I've often heard of the saddle-point method but never got around to learning it (e.g. didn't get that far in the Analytic Combinatorics book); now I have some idea. So if I understand this answer correctly, this gives the asymptotics of the largest coefficient... and we need to appeal to probability arguments etc to show that for the other coefficients, the probability drops off exponentially from this main (largest) term.
    $endgroup$
    – ShreevatsaR
    Jan 15 at 19:25












  • $begingroup$
    @ShreevatsaR: You're welcome and I agree with your considerations. I'd like to point to the introductory paragraph (the first one) in section VIII.9: Saddle-points and probability distributions. This section might go in the directions you are interested in.
    $endgroup$
    – Markus Scheuer
    Jan 15 at 19:36




















  • $begingroup$
    Thank you, those are useful references!
    $endgroup$
    – ShreevatsaR
    Jan 12 at 16:56










  • $begingroup$
    @ShreevatsaR: You're welcome.
    $endgroup$
    – Markus Scheuer
    Jan 12 at 16:58










  • $begingroup$
    Thanks very much for a very clear and informative answer. I've often heard of the saddle-point method but never got around to learning it (e.g. didn't get that far in the Analytic Combinatorics book); now I have some idea. So if I understand this answer correctly, this gives the asymptotics of the largest coefficient... and we need to appeal to probability arguments etc to show that for the other coefficients, the probability drops off exponentially from this main (largest) term.
    $endgroup$
    – ShreevatsaR
    Jan 15 at 19:25












  • $begingroup$
    @ShreevatsaR: You're welcome and I agree with your considerations. I'd like to point to the introductory paragraph (the first one) in section VIII.9: Saddle-points and probability distributions. This section might go in the directions you are interested in.
    $endgroup$
    – Markus Scheuer
    Jan 15 at 19:36


















$begingroup$
Thank you, those are useful references!
$endgroup$
– ShreevatsaR
Jan 12 at 16:56




$begingroup$
Thank you, those are useful references!
$endgroup$
– ShreevatsaR
Jan 12 at 16:56












$begingroup$
@ShreevatsaR: You're welcome.
$endgroup$
– Markus Scheuer
Jan 12 at 16:58




$begingroup$
@ShreevatsaR: You're welcome.
$endgroup$
– Markus Scheuer
Jan 12 at 16:58












$begingroup$
Thanks very much for a very clear and informative answer. I've often heard of the saddle-point method but never got around to learning it (e.g. didn't get that far in the Analytic Combinatorics book); now I have some idea. So if I understand this answer correctly, this gives the asymptotics of the largest coefficient... and we need to appeal to probability arguments etc to show that for the other coefficients, the probability drops off exponentially from this main (largest) term.
$endgroup$
– ShreevatsaR
Jan 15 at 19:25






$begingroup$
Thanks very much for a very clear and informative answer. I've often heard of the saddle-point method but never got around to learning it (e.g. didn't get that far in the Analytic Combinatorics book); now I have some idea. So if I understand this answer correctly, this gives the asymptotics of the largest coefficient... and we need to appeal to probability arguments etc to show that for the other coefficients, the probability drops off exponentially from this main (largest) term.
$endgroup$
– ShreevatsaR
Jan 15 at 19:25














$begingroup$
@ShreevatsaR: You're welcome and I agree with your considerations. I'd like to point to the introductory paragraph (the first one) in section VIII.9: Saddle-points and probability distributions. This section might go in the directions you are interested in.
$endgroup$
– Markus Scheuer
Jan 15 at 19:36






$begingroup$
@ShreevatsaR: You're welcome and I agree with your considerations. I'd like to point to the introductory paragraph (the first one) in section VIII.9: Saddle-points and probability distributions. This section might go in the directions you are interested in.
$endgroup$
– Markus Scheuer
Jan 15 at 19:36













3












$begingroup$

You can prove that your asymptotic expression is correct using the Edgeworth series.



Let $F_k$ be the cdf for $sqrt{frac{k}{8.25}}(S_k/k-4.5)$. By the Central Limit theorem, $F_k(x)$ is approximately equal to $Phi(x)$. Specifically, the Edgeworth series shows that
$$
F_k(x)=Phi(x) -frac{lambda_3}{6sqrt k}Phi'''(x)+O(k^{-1})
$$

Here, $lambda_3$ is the skewness of a single random digit. Since this skewness is zero, as the distribution is symmetric about $[0,9]$, we get
$$
F_k(x)=Phi(x)+O(k^{-1}).
$$

This shows the error in the central limit approximation is linear in $k$. Therefore,
begin{align}
P(S_k=n)
&=P(n-tfrac12<S_kle n+tfrac12)
\&=F_kleft(frac{n+frac12-4.5k}{sqrt{8.25 k}}right)-F_kleft(frac{n-frac12-4.5k}{sqrt{8.25 k}}right)
\&=Phileft(frac{n+frac12-4.5k}{sqrt{8.25 k}}right)-Phileft(frac{n-frac12-4.5k}{sqrt{8.25 k}}right)+O(k^{-1})
\&stackrel1= Phi'(xi)cdotfrac1{sqrt{8.25k}}+O(k^{-1})
\&= frac1{sqrt{8.25k}}left(Phi'(xi)-Phi'left(frac{n-4.5k}{sqrt{8.25k}}right)+Phi'left(frac{n-4.5k}{sqrt{8.25k}}right)right)+O(k^{-1})
\&stackrel2= frac1{sqrt{8.25k}}Phi'left(frac{n-4.5k}{sqrt{8.25k}}right)+frac1{8.25k}Phi''(eta)+O(k^{-1})
\&stackrel3= frac1{sqrt{8.25k}}Phi'left(frac{n-4.5k}{sqrt{8.25k}}right)+O(k^{-1})
end{align}



Explanations:




  1. Here, we are applying the mean value theorem. $xi$ is a number between $frac{npmfrac12-4.5k}{sqrt{8.25 k}}$.


  2. Now we apply the mean value theorem to $Phi'$. Here, $eta$ is a number between $xi$ and $frac{n-4.5k}{sqrt{8.25 k}}$


  3. We can absorb the $frac1{8.25k}Phi''(eta)$ into the $O(k^{-1})$ because $Phi''$ is bounded.



This is exactly the asymptotic expansion you guessed; however, the above rigorously shows that your answer is correct, with the error decreasing linearly as $ktoinfty$.






share|cite|improve this answer









$endgroup$













  • $begingroup$
    Actually, this is omitting some details; in order for the Edgeworth series to work when the variables are integer valued, you need to add in a correction term which is periodic, as described in jstor.org/stable/2242145?seq=1#metadata_info_tab_contents. This term ends up canceling out; I will add in more details later.
    $endgroup$
    – Mike Earnest
    Jan 12 at 20:03










  • $begingroup$
    Wow these look like powerful techniques; thanks for sharing.
    $endgroup$
    – ShreevatsaR
    Jan 12 at 20:16










  • $begingroup$
    @ShreevatsaR I said I would post more info, but I have tried and been unable to figure out the correct result. I would suggest you try to look into the Edgeworth series, and specifically the series for lattice variables, and see if you can figure it out.
    $endgroup$
    – Mike Earnest
    Jan 14 at 23:40










  • $begingroup$
    Sure, thanks for your effort and for a very useful lead, and what matters is that I learned something new and useful... yes I'll try to look into this further too.
    $endgroup$
    – ShreevatsaR
    Jan 15 at 18:45
















3












$begingroup$

You can prove that your asymptotic expression is correct using the Edgeworth series.



Let $F_k$ be the cdf for $sqrt{frac{k}{8.25}}(S_k/k-4.5)$. By the Central Limit theorem, $F_k(x)$ is approximately equal to $Phi(x)$. Specifically, the Edgeworth series shows that
$$
F_k(x)=Phi(x) -frac{lambda_3}{6sqrt k}Phi'''(x)+O(k^{-1})
$$

Here, $lambda_3$ is the skewness of a single random digit. Since this skewness is zero, as the distribution is symmetric about $[0,9]$, we get
$$
F_k(x)=Phi(x)+O(k^{-1}).
$$

This shows the error in the central limit approximation is linear in $k$. Therefore,
begin{align}
P(S_k=n)
&=P(n-tfrac12<S_kle n+tfrac12)
\&=F_kleft(frac{n+frac12-4.5k}{sqrt{8.25 k}}right)-F_kleft(frac{n-frac12-4.5k}{sqrt{8.25 k}}right)
\&=Phileft(frac{n+frac12-4.5k}{sqrt{8.25 k}}right)-Phileft(frac{n-frac12-4.5k}{sqrt{8.25 k}}right)+O(k^{-1})
\&stackrel1= Phi'(xi)cdotfrac1{sqrt{8.25k}}+O(k^{-1})
\&= frac1{sqrt{8.25k}}left(Phi'(xi)-Phi'left(frac{n-4.5k}{sqrt{8.25k}}right)+Phi'left(frac{n-4.5k}{sqrt{8.25k}}right)right)+O(k^{-1})
\&stackrel2= frac1{sqrt{8.25k}}Phi'left(frac{n-4.5k}{sqrt{8.25k}}right)+frac1{8.25k}Phi''(eta)+O(k^{-1})
\&stackrel3= frac1{sqrt{8.25k}}Phi'left(frac{n-4.5k}{sqrt{8.25k}}right)+O(k^{-1})
end{align}



Explanations:




  1. Here, we are applying the mean value theorem. $xi$ is a number between $frac{npmfrac12-4.5k}{sqrt{8.25 k}}$.


  2. Now we apply the mean value theorem to $Phi'$. Here, $eta$ is a number between $xi$ and $frac{n-4.5k}{sqrt{8.25 k}}$


  3. We can absorb the $frac1{8.25k}Phi''(eta)$ into the $O(k^{-1})$ because $Phi''$ is bounded.



This is exactly the asymptotic expansion you guessed; however, the above rigorously shows that your answer is correct, with the error decreasing linearly as $ktoinfty$.






share|cite|improve this answer









$endgroup$













  • $begingroup$
    Actually, this is omitting some details; in order for the Edgeworth series to work when the variables are integer valued, you need to add in a correction term which is periodic, as described in jstor.org/stable/2242145?seq=1#metadata_info_tab_contents. This term ends up canceling out; I will add in more details later.
    $endgroup$
    – Mike Earnest
    Jan 12 at 20:03










  • $begingroup$
    Wow these look like powerful techniques; thanks for sharing.
    $endgroup$
    – ShreevatsaR
    Jan 12 at 20:16










  • $begingroup$
    @ShreevatsaR I said I would post more info, but I have tried and been unable to figure out the correct result. I would suggest you try to look into the Edgeworth series, and specifically the series for lattice variables, and see if you can figure it out.
    $endgroup$
    – Mike Earnest
    Jan 14 at 23:40










  • $begingroup$
    Sure, thanks for your effort and for a very useful lead, and what matters is that I learned something new and useful... yes I'll try to look into this further too.
    $endgroup$
    – ShreevatsaR
    Jan 15 at 18:45














3












3








3





$begingroup$

You can prove that your asymptotic expression is correct using the Edgeworth series.



Let $F_k$ be the cdf for $sqrt{frac{k}{8.25}}(S_k/k-4.5)$. By the Central Limit theorem, $F_k(x)$ is approximately equal to $Phi(x)$. Specifically, the Edgeworth series shows that
$$
F_k(x)=Phi(x) -frac{lambda_3}{6sqrt k}Phi'''(x)+O(k^{-1})
$$

Here, $lambda_3$ is the skewness of a single random digit. Since this skewness is zero, as the distribution is symmetric about $[0,9]$, we get
$$
F_k(x)=Phi(x)+O(k^{-1}).
$$

This shows the error in the central limit approximation is linear in $k$. Therefore,
begin{align}
P(S_k=n)
&=P(n-tfrac12<S_kle n+tfrac12)
\&=F_kleft(frac{n+frac12-4.5k}{sqrt{8.25 k}}right)-F_kleft(frac{n-frac12-4.5k}{sqrt{8.25 k}}right)
\&=Phileft(frac{n+frac12-4.5k}{sqrt{8.25 k}}right)-Phileft(frac{n-frac12-4.5k}{sqrt{8.25 k}}right)+O(k^{-1})
\&stackrel1= Phi'(xi)cdotfrac1{sqrt{8.25k}}+O(k^{-1})
\&= frac1{sqrt{8.25k}}left(Phi'(xi)-Phi'left(frac{n-4.5k}{sqrt{8.25k}}right)+Phi'left(frac{n-4.5k}{sqrt{8.25k}}right)right)+O(k^{-1})
\&stackrel2= frac1{sqrt{8.25k}}Phi'left(frac{n-4.5k}{sqrt{8.25k}}right)+frac1{8.25k}Phi''(eta)+O(k^{-1})
\&stackrel3= frac1{sqrt{8.25k}}Phi'left(frac{n-4.5k}{sqrt{8.25k}}right)+O(k^{-1})
end{align}



Explanations:




  1. Here, we are applying the mean value theorem. $xi$ is a number between $frac{npmfrac12-4.5k}{sqrt{8.25 k}}$.


  2. Now we apply the mean value theorem to $Phi'$. Here, $eta$ is a number between $xi$ and $frac{n-4.5k}{sqrt{8.25 k}}$


  3. We can absorb the $frac1{8.25k}Phi''(eta)$ into the $O(k^{-1})$ because $Phi''$ is bounded.



This is exactly the asymptotic expansion you guessed; however, the above rigorously shows that your answer is correct, with the error decreasing linearly as $ktoinfty$.






share|cite|improve this answer









$endgroup$



You can prove that your asymptotic expression is correct using the Edgeworth series.



Let $F_k$ be the cdf for $sqrt{frac{k}{8.25}}(S_k/k-4.5)$. By the Central Limit theorem, $F_k(x)$ is approximately equal to $Phi(x)$. Specifically, the Edgeworth series shows that
$$
F_k(x)=Phi(x) -frac{lambda_3}{6sqrt k}Phi'''(x)+O(k^{-1})
$$

Here, $lambda_3$ is the skewness of a single random digit. Since this skewness is zero, as the distribution is symmetric about $[0,9]$, we get
$$
F_k(x)=Phi(x)+O(k^{-1}).
$$

This shows the error in the central limit approximation is linear in $k$. Therefore,
begin{align}
P(S_k=n)
&=P(n-tfrac12<S_kle n+tfrac12)
\&=F_kleft(frac{n+frac12-4.5k}{sqrt{8.25 k}}right)-F_kleft(frac{n-frac12-4.5k}{sqrt{8.25 k}}right)
\&=Phileft(frac{n+frac12-4.5k}{sqrt{8.25 k}}right)-Phileft(frac{n-frac12-4.5k}{sqrt{8.25 k}}right)+O(k^{-1})
\&stackrel1= Phi'(xi)cdotfrac1{sqrt{8.25k}}+O(k^{-1})
\&= frac1{sqrt{8.25k}}left(Phi'(xi)-Phi'left(frac{n-4.5k}{sqrt{8.25k}}right)+Phi'left(frac{n-4.5k}{sqrt{8.25k}}right)right)+O(k^{-1})
\&stackrel2= frac1{sqrt{8.25k}}Phi'left(frac{n-4.5k}{sqrt{8.25k}}right)+frac1{8.25k}Phi''(eta)+O(k^{-1})
\&stackrel3= frac1{sqrt{8.25k}}Phi'left(frac{n-4.5k}{sqrt{8.25k}}right)+O(k^{-1})
end{align}



Explanations:




  1. Here, we are applying the mean value theorem. $xi$ is a number between $frac{npmfrac12-4.5k}{sqrt{8.25 k}}$.


  2. Now we apply the mean value theorem to $Phi'$. Here, $eta$ is a number between $xi$ and $frac{n-4.5k}{sqrt{8.25 k}}$


  3. We can absorb the $frac1{8.25k}Phi''(eta)$ into the $O(k^{-1})$ because $Phi''$ is bounded.



This is exactly the asymptotic expansion you guessed; however, the above rigorously shows that your answer is correct, with the error decreasing linearly as $ktoinfty$.







share|cite|improve this answer












share|cite|improve this answer



share|cite|improve this answer










answered Jan 12 at 19:22









Mike EarnestMike Earnest

27.3k22152




27.3k22152












  • $begingroup$
    Actually, this is omitting some details; in order for the Edgeworth series to work when the variables are integer valued, you need to add in a correction term which is periodic, as described in jstor.org/stable/2242145?seq=1#metadata_info_tab_contents. This term ends up canceling out; I will add in more details later.
    $endgroup$
    – Mike Earnest
    Jan 12 at 20:03










  • $begingroup$
    Wow these look like powerful techniques; thanks for sharing.
    $endgroup$
    – ShreevatsaR
    Jan 12 at 20:16










  • $begingroup$
    @ShreevatsaR I said I would post more info, but I have tried and been unable to figure out the correct result. I would suggest you try to look into the Edgeworth series, and specifically the series for lattice variables, and see if you can figure it out.
    $endgroup$
    – Mike Earnest
    Jan 14 at 23:40










  • $begingroup$
    Sure, thanks for your effort and for a very useful lead, and what matters is that I learned something new and useful... yes I'll try to look into this further too.
    $endgroup$
    – ShreevatsaR
    Jan 15 at 18:45


















  • $begingroup$
    Actually, this is omitting some details; in order for the Edgeworth series to work when the variables are integer valued, you need to add in a correction term which is periodic, as described in jstor.org/stable/2242145?seq=1#metadata_info_tab_contents. This term ends up canceling out; I will add in more details later.
    $endgroup$
    – Mike Earnest
    Jan 12 at 20:03










  • $begingroup$
    Wow these look like powerful techniques; thanks for sharing.
    $endgroup$
    – ShreevatsaR
    Jan 12 at 20:16










  • $begingroup$
    @ShreevatsaR I said I would post more info, but I have tried and been unable to figure out the correct result. I would suggest you try to look into the Edgeworth series, and specifically the series for lattice variables, and see if you can figure it out.
    $endgroup$
    – Mike Earnest
    Jan 14 at 23:40










  • $begingroup$
    Sure, thanks for your effort and for a very useful lead, and what matters is that I learned something new and useful... yes I'll try to look into this further too.
    $endgroup$
    – ShreevatsaR
    Jan 15 at 18:45
















$begingroup$
Actually, this is omitting some details; in order for the Edgeworth series to work when the variables are integer valued, you need to add in a correction term which is periodic, as described in jstor.org/stable/2242145?seq=1#metadata_info_tab_contents. This term ends up canceling out; I will add in more details later.
$endgroup$
– Mike Earnest
Jan 12 at 20:03




$begingroup$
Actually, this is omitting some details; in order for the Edgeworth series to work when the variables are integer valued, you need to add in a correction term which is periodic, as described in jstor.org/stable/2242145?seq=1#metadata_info_tab_contents. This term ends up canceling out; I will add in more details later.
$endgroup$
– Mike Earnest
Jan 12 at 20:03












$begingroup$
Wow these look like powerful techniques; thanks for sharing.
$endgroup$
– ShreevatsaR
Jan 12 at 20:16




$begingroup$
Wow these look like powerful techniques; thanks for sharing.
$endgroup$
– ShreevatsaR
Jan 12 at 20:16












$begingroup$
@ShreevatsaR I said I would post more info, but I have tried and been unable to figure out the correct result. I would suggest you try to look into the Edgeworth series, and specifically the series for lattice variables, and see if you can figure it out.
$endgroup$
– Mike Earnest
Jan 14 at 23:40




$begingroup$
@ShreevatsaR I said I would post more info, but I have tried and been unable to figure out the correct result. I would suggest you try to look into the Edgeworth series, and specifically the series for lattice variables, and see if you can figure it out.
$endgroup$
– Mike Earnest
Jan 14 at 23:40












$begingroup$
Sure, thanks for your effort and for a very useful lead, and what matters is that I learned something new and useful... yes I'll try to look into this further too.
$endgroup$
– ShreevatsaR
Jan 15 at 18:45




$begingroup$
Sure, thanks for your effort and for a very useful lead, and what matters is that I learned something new and useful... yes I'll try to look into this further too.
$endgroup$
– ShreevatsaR
Jan 15 at 18:45











2





+200







$begingroup$

Each digit in a $(r+1)$-ary base is a discrete uniform random variable, with support $[0,r]$.

The relevant mean and variance are
$$
mu = {r over 2}quad sigma ^{,2} = {{left( {r + 1} right)^{,2} - 1} over {12}}
$$



By the Central Limit Theorem the sum$n$ of $k$ of them will tend to be Normally distributed
with mean $k mu$ and variance $k sigma ^2$.

That is the expression that you give will tend (very quickly) to
$$
{1 over {sqrt {2pi ksigma ^{,2} } }}e^{, - ,{{left( {n - kmu } right)^{,2} } over {2ksigma ^{,2} }}}
= {{sqrt {6/pi } } over {sqrt {k,rleft( {r + 2} right)} }}e^{, - ,6{{left( {n - kr/2} right)^{,2} } over {k,rleft( {r + 2} right)}}}
$$



Then just replace $r=9$.



That for what concerns the asymptotic distribution.



Concerning your other questions, I did not understood properly what you want to compute.



------ Addendum in reply to your comment ------



1) A discrete uniform variable $0 le n le r$ is approximable to a continuous uniform
variable $-1/2 le nu le r+1/2$, with $p(n) approx p(n-1/2 le nu le n+1/2)$.



2) If $p(n;;,r,,k)$ denotes the probability above (either "exact" or "approximated") to get $n$ as the sum of $k$ $(r+1)$-ary digits (i.i.d.),
then its complement is
$$
q(n;;,r,,k) = 1 - p(n;;,r,,k)
$$

So, the probability that $n$ is not attained as the sum of either $1$, or $2$, .., or $m$ digits will be
$$
Q(n;;,r,,m) = prodlimits_{1, le ,k, le ,m} {q(n;;,r,,k)} = prodlimits_{1, le ,k, le ,m} {left( {1 - p(n;;,r,,k)} right)}
$$

For small $p(n;;,r,,k)$ (i.e. high $r,k$) we can approximate the above as
$$
eqalign{
& ln Q(n;;,r,,m) = sumlimits_{1, le ,k, le ,m} {ln left( {1 - p(n;;,r,,k)} right)} approx cr
& approx - sumlimits_{1, le ,k, le ,m} {p(n;;,r,,k) + Oleft( {p(n;;,r,,k)^{,2} } right)} cr}
$$



3) Finally note that the exact $p(n;;,r,,k)$ can be better written as
$$
eqalign{
& p(n;;,r,,k) = {{N_{,b} (n,r,k)} over {left( {r + 1} right)^{,m} }} = cr
& = sumlimits_{left( {0, le } right),,j,,left( { le ,{n over {r + 1}}, le ,k} right)} {left( { - 1} right)^j binom{k}{j}
binom{n + k - 1 - jleft( {r + 1} right)}{ n - jleft( {r + 1} right)}
} cr}
$$



which, as thoroughly explained in this related post, gives the advantage that the sum limits are implicit in the binomial coefficient,
thereby simplifying its algebraic manipulation.






share|cite|improve this answer











$endgroup$













  • $begingroup$
    Thanks… (1) can you explain how you deal with the fact that the normal distribution is a continuous one and we're asking about the sum being exactly one $n$ (a point)? (2) My second question was about summing the above over all integers $k$. In other words, first we take the sum $S_1$ of one random digit, then the sum $S_2$ of two random digits, then the sum $S_3$ of three random digits and so on, and ask what is the probability that $n$ is a member of this sequence. We have one normal distribution for each $k$, and we want the total probability distribution that falls on $n$.
    $endgroup$
    – ShreevatsaR
    Jan 7 at 3:57










  • $begingroup$
    @ShreevatsaR: I expanded my answer in reply to your comment: does it meet your requirements ?
    $endgroup$
    – G Cab
    Jan 7 at 17:21










  • $begingroup$
    Thank you very much (+1). I'll think about this a bit more but I think the initial part matches. BTW for the second part, I was interested in the case where (in your notation) $m to infty$ -- note that as $k$ becomes larger than than some threshold, again the probability of hitting $n$ starts falling off exponentially, so it should be possible to sum over all $k$ and get an expression $Q(n; r)$ (in your notation) that depends only on $n$ (and $r$). Maybe not even $n$...
    $endgroup$
    – ShreevatsaR
    Jan 7 at 18:36










  • $begingroup$
    increasing $k$ increases the mean of the gaussian $p(n,r,k)$, while the std dev. increases with $sqrt{k}$. So, keeping $n$ fixed, the gaussian will move towards and then beyond it, with an $O(exp(-k)/ sqrt{k})$ which is certainly summable.
    $endgroup$
    – G Cab
    Jan 7 at 19:22










  • $begingroup$
    Yes exactly; that's what I expect and that's why I'd like to know asymptotically what the sum is equal to. Maybe I'll post a separate question about how to sum it over all $k$. :-) (and accept this one after a while).
    $endgroup$
    – ShreevatsaR
    Jan 7 at 20:25
















2





+200







$begingroup$

Each digit in a $(r+1)$-ary base is a discrete uniform random variable, with support $[0,r]$.

The relevant mean and variance are
$$
mu = {r over 2}quad sigma ^{,2} = {{left( {r + 1} right)^{,2} - 1} over {12}}
$$



By the Central Limit Theorem the sum$n$ of $k$ of them will tend to be Normally distributed
with mean $k mu$ and variance $k sigma ^2$.

That is the expression that you give will tend (very quickly) to
$$
{1 over {sqrt {2pi ksigma ^{,2} } }}e^{, - ,{{left( {n - kmu } right)^{,2} } over {2ksigma ^{,2} }}}
= {{sqrt {6/pi } } over {sqrt {k,rleft( {r + 2} right)} }}e^{, - ,6{{left( {n - kr/2} right)^{,2} } over {k,rleft( {r + 2} right)}}}
$$



Then just replace $r=9$.



That for what concerns the asymptotic distribution.



Concerning your other questions, I did not understood properly what you want to compute.



------ Addendum in reply to your comment ------



1) A discrete uniform variable $0 le n le r$ is approximable to a continuous uniform
variable $-1/2 le nu le r+1/2$, with $p(n) approx p(n-1/2 le nu le n+1/2)$.



2) If $p(n;;,r,,k)$ denotes the probability above (either "exact" or "approximated") to get $n$ as the sum of $k$ $(r+1)$-ary digits (i.i.d.),
then its complement is
$$
q(n;;,r,,k) = 1 - p(n;;,r,,k)
$$

So, the probability that $n$ is not attained as the sum of either $1$, or $2$, .., or $m$ digits will be
$$
Q(n;;,r,,m) = prodlimits_{1, le ,k, le ,m} {q(n;;,r,,k)} = prodlimits_{1, le ,k, le ,m} {left( {1 - p(n;;,r,,k)} right)}
$$

For small $p(n;;,r,,k)$ (i.e. high $r,k$) we can approximate the above as
$$
eqalign{
& ln Q(n;;,r,,m) = sumlimits_{1, le ,k, le ,m} {ln left( {1 - p(n;;,r,,k)} right)} approx cr
& approx - sumlimits_{1, le ,k, le ,m} {p(n;;,r,,k) + Oleft( {p(n;;,r,,k)^{,2} } right)} cr}
$$



3) Finally note that the exact $p(n;;,r,,k)$ can be better written as
$$
eqalign{
& p(n;;,r,,k) = {{N_{,b} (n,r,k)} over {left( {r + 1} right)^{,m} }} = cr
& = sumlimits_{left( {0, le } right),,j,,left( { le ,{n over {r + 1}}, le ,k} right)} {left( { - 1} right)^j binom{k}{j}
binom{n + k - 1 - jleft( {r + 1} right)}{ n - jleft( {r + 1} right)}
} cr}
$$



which, as thoroughly explained in this related post, gives the advantage that the sum limits are implicit in the binomial coefficient,
thereby simplifying its algebraic manipulation.






share|cite|improve this answer











$endgroup$













  • $begingroup$
    Thanks… (1) can you explain how you deal with the fact that the normal distribution is a continuous one and we're asking about the sum being exactly one $n$ (a point)? (2) My second question was about summing the above over all integers $k$. In other words, first we take the sum $S_1$ of one random digit, then the sum $S_2$ of two random digits, then the sum $S_3$ of three random digits and so on, and ask what is the probability that $n$ is a member of this sequence. We have one normal distribution for each $k$, and we want the total probability distribution that falls on $n$.
    $endgroup$
    – ShreevatsaR
    Jan 7 at 3:57










  • $begingroup$
    @ShreevatsaR: I expanded my answer in reply to your comment: does it meet your requirements ?
    $endgroup$
    – G Cab
    Jan 7 at 17:21










  • $begingroup$
    Thank you very much (+1). I'll think about this a bit more but I think the initial part matches. BTW for the second part, I was interested in the case where (in your notation) $m to infty$ -- note that as $k$ becomes larger than than some threshold, again the probability of hitting $n$ starts falling off exponentially, so it should be possible to sum over all $k$ and get an expression $Q(n; r)$ (in your notation) that depends only on $n$ (and $r$). Maybe not even $n$...
    $endgroup$
    – ShreevatsaR
    Jan 7 at 18:36










  • $begingroup$
    increasing $k$ increases the mean of the gaussian $p(n,r,k)$, while the std dev. increases with $sqrt{k}$. So, keeping $n$ fixed, the gaussian will move towards and then beyond it, with an $O(exp(-k)/ sqrt{k})$ which is certainly summable.
    $endgroup$
    – G Cab
    Jan 7 at 19:22










  • $begingroup$
    Yes exactly; that's what I expect and that's why I'd like to know asymptotically what the sum is equal to. Maybe I'll post a separate question about how to sum it over all $k$. :-) (and accept this one after a while).
    $endgroup$
    – ShreevatsaR
    Jan 7 at 20:25














2





+200







2





+200



2




+200



$begingroup$

Each digit in a $(r+1)$-ary base is a discrete uniform random variable, with support $[0,r]$.

The relevant mean and variance are
$$
mu = {r over 2}quad sigma ^{,2} = {{left( {r + 1} right)^{,2} - 1} over {12}}
$$



By the Central Limit Theorem the sum$n$ of $k$ of them will tend to be Normally distributed
with mean $k mu$ and variance $k sigma ^2$.

That is the expression that you give will tend (very quickly) to
$$
{1 over {sqrt {2pi ksigma ^{,2} } }}e^{, - ,{{left( {n - kmu } right)^{,2} } over {2ksigma ^{,2} }}}
= {{sqrt {6/pi } } over {sqrt {k,rleft( {r + 2} right)} }}e^{, - ,6{{left( {n - kr/2} right)^{,2} } over {k,rleft( {r + 2} right)}}}
$$



Then just replace $r=9$.



That for what concerns the asymptotic distribution.



Concerning your other questions, I did not understood properly what you want to compute.



------ Addendum in reply to your comment ------



1) A discrete uniform variable $0 le n le r$ is approximable to a continuous uniform
variable $-1/2 le nu le r+1/2$, with $p(n) approx p(n-1/2 le nu le n+1/2)$.



2) If $p(n;;,r,,k)$ denotes the probability above (either "exact" or "approximated") to get $n$ as the sum of $k$ $(r+1)$-ary digits (i.i.d.),
then its complement is
$$
q(n;;,r,,k) = 1 - p(n;;,r,,k)
$$

So, the probability that $n$ is not attained as the sum of either $1$, or $2$, .., or $m$ digits will be
$$
Q(n;;,r,,m) = prodlimits_{1, le ,k, le ,m} {q(n;;,r,,k)} = prodlimits_{1, le ,k, le ,m} {left( {1 - p(n;;,r,,k)} right)}
$$

For small $p(n;;,r,,k)$ (i.e. high $r,k$) we can approximate the above as
$$
eqalign{
& ln Q(n;;,r,,m) = sumlimits_{1, le ,k, le ,m} {ln left( {1 - p(n;;,r,,k)} right)} approx cr
& approx - sumlimits_{1, le ,k, le ,m} {p(n;;,r,,k) + Oleft( {p(n;;,r,,k)^{,2} } right)} cr}
$$



3) Finally note that the exact $p(n;;,r,,k)$ can be better written as
$$
eqalign{
& p(n;;,r,,k) = {{N_{,b} (n,r,k)} over {left( {r + 1} right)^{,m} }} = cr
& = sumlimits_{left( {0, le } right),,j,,left( { le ,{n over {r + 1}}, le ,k} right)} {left( { - 1} right)^j binom{k}{j}
binom{n + k - 1 - jleft( {r + 1} right)}{ n - jleft( {r + 1} right)}
} cr}
$$



which, as thoroughly explained in this related post, gives the advantage that the sum limits are implicit in the binomial coefficient,
thereby simplifying its algebraic manipulation.






share|cite|improve this answer











$endgroup$



Each digit in a $(r+1)$-ary base is a discrete uniform random variable, with support $[0,r]$.

The relevant mean and variance are
$$
mu = {r over 2}quad sigma ^{,2} = {{left( {r + 1} right)^{,2} - 1} over {12}}
$$



By the Central Limit Theorem the sum$n$ of $k$ of them will tend to be Normally distributed
with mean $k mu$ and variance $k sigma ^2$.

That is the expression that you give will tend (very quickly) to
$$
{1 over {sqrt {2pi ksigma ^{,2} } }}e^{, - ,{{left( {n - kmu } right)^{,2} } over {2ksigma ^{,2} }}}
= {{sqrt {6/pi } } over {sqrt {k,rleft( {r + 2} right)} }}e^{, - ,6{{left( {n - kr/2} right)^{,2} } over {k,rleft( {r + 2} right)}}}
$$



Then just replace $r=9$.



That for what concerns the asymptotic distribution.



Concerning your other questions, I did not understood properly what you want to compute.



------ Addendum in reply to your comment ------



1) A discrete uniform variable $0 le n le r$ is approximable to a continuous uniform
variable $-1/2 le nu le r+1/2$, with $p(n) approx p(n-1/2 le nu le n+1/2)$.



2) If $p(n;;,r,,k)$ denotes the probability above (either "exact" or "approximated") to get $n$ as the sum of $k$ $(r+1)$-ary digits (i.i.d.),
then its complement is
$$
q(n;;,r,,k) = 1 - p(n;;,r,,k)
$$

So, the probability that $n$ is not attained as the sum of either $1$, or $2$, .., or $m$ digits will be
$$
Q(n;;,r,,m) = prodlimits_{1, le ,k, le ,m} {q(n;;,r,,k)} = prodlimits_{1, le ,k, le ,m} {left( {1 - p(n;;,r,,k)} right)}
$$

For small $p(n;;,r,,k)$ (i.e. high $r,k$) we can approximate the above as
$$
eqalign{
& ln Q(n;;,r,,m) = sumlimits_{1, le ,k, le ,m} {ln left( {1 - p(n;;,r,,k)} right)} approx cr
& approx - sumlimits_{1, le ,k, le ,m} {p(n;;,r,,k) + Oleft( {p(n;;,r,,k)^{,2} } right)} cr}
$$



3) Finally note that the exact $p(n;;,r,,k)$ can be better written as
$$
eqalign{
& p(n;;,r,,k) = {{N_{,b} (n,r,k)} over {left( {r + 1} right)^{,m} }} = cr
& = sumlimits_{left( {0, le } right),,j,,left( { le ,{n over {r + 1}}, le ,k} right)} {left( { - 1} right)^j binom{k}{j}
binom{n + k - 1 - jleft( {r + 1} right)}{ n - jleft( {r + 1} right)}
} cr}
$$



which, as thoroughly explained in this related post, gives the advantage that the sum limits are implicit in the binomial coefficient,
thereby simplifying its algebraic manipulation.







share|cite|improve this answer














share|cite|improve this answer



share|cite|improve this answer








edited Jan 7 at 22:19

























answered Jan 6 at 23:59









G CabG Cab

20.4k31341




20.4k31341












  • $begingroup$
    Thanks… (1) can you explain how you deal with the fact that the normal distribution is a continuous one and we're asking about the sum being exactly one $n$ (a point)? (2) My second question was about summing the above over all integers $k$. In other words, first we take the sum $S_1$ of one random digit, then the sum $S_2$ of two random digits, then the sum $S_3$ of three random digits and so on, and ask what is the probability that $n$ is a member of this sequence. We have one normal distribution for each $k$, and we want the total probability distribution that falls on $n$.
    $endgroup$
    – ShreevatsaR
    Jan 7 at 3:57










  • $begingroup$
    @ShreevatsaR: I expanded my answer in reply to your comment: does it meet your requirements ?
    $endgroup$
    – G Cab
    Jan 7 at 17:21










  • $begingroup$
    Thank you very much (+1). I'll think about this a bit more but I think the initial part matches. BTW for the second part, I was interested in the case where (in your notation) $m to infty$ -- note that as $k$ becomes larger than than some threshold, again the probability of hitting $n$ starts falling off exponentially, so it should be possible to sum over all $k$ and get an expression $Q(n; r)$ (in your notation) that depends only on $n$ (and $r$). Maybe not even $n$...
    $endgroup$
    – ShreevatsaR
    Jan 7 at 18:36










  • $begingroup$
    increasing $k$ increases the mean of the gaussian $p(n,r,k)$, while the std dev. increases with $sqrt{k}$. So, keeping $n$ fixed, the gaussian will move towards and then beyond it, with an $O(exp(-k)/ sqrt{k})$ which is certainly summable.
    $endgroup$
    – G Cab
    Jan 7 at 19:22










  • $begingroup$
    Yes exactly; that's what I expect and that's why I'd like to know asymptotically what the sum is equal to. Maybe I'll post a separate question about how to sum it over all $k$. :-) (and accept this one after a while).
    $endgroup$
    – ShreevatsaR
    Jan 7 at 20:25


















  • $begingroup$
    Thanks… (1) can you explain how you deal with the fact that the normal distribution is a continuous one and we're asking about the sum being exactly one $n$ (a point)? (2) My second question was about summing the above over all integers $k$. In other words, first we take the sum $S_1$ of one random digit, then the sum $S_2$ of two random digits, then the sum $S_3$ of three random digits and so on, and ask what is the probability that $n$ is a member of this sequence. We have one normal distribution for each $k$, and we want the total probability distribution that falls on $n$.
    $endgroup$
    – ShreevatsaR
    Jan 7 at 3:57










  • $begingroup$
    @ShreevatsaR: I expanded my answer in reply to your comment: does it meet your requirements ?
    $endgroup$
    – G Cab
    Jan 7 at 17:21










  • $begingroup$
    Thank you very much (+1). I'll think about this a bit more but I think the initial part matches. BTW for the second part, I was interested in the case where (in your notation) $m to infty$ -- note that as $k$ becomes larger than than some threshold, again the probability of hitting $n$ starts falling off exponentially, so it should be possible to sum over all $k$ and get an expression $Q(n; r)$ (in your notation) that depends only on $n$ (and $r$). Maybe not even $n$...
    $endgroup$
    – ShreevatsaR
    Jan 7 at 18:36










  • $begingroup$
    increasing $k$ increases the mean of the gaussian $p(n,r,k)$, while the std dev. increases with $sqrt{k}$. So, keeping $n$ fixed, the gaussian will move towards and then beyond it, with an $O(exp(-k)/ sqrt{k})$ which is certainly summable.
    $endgroup$
    – G Cab
    Jan 7 at 19:22










  • $begingroup$
    Yes exactly; that's what I expect and that's why I'd like to know asymptotically what the sum is equal to. Maybe I'll post a separate question about how to sum it over all $k$. :-) (and accept this one after a while).
    $endgroup$
    – ShreevatsaR
    Jan 7 at 20:25
















$begingroup$
Thanks… (1) can you explain how you deal with the fact that the normal distribution is a continuous one and we're asking about the sum being exactly one $n$ (a point)? (2) My second question was about summing the above over all integers $k$. In other words, first we take the sum $S_1$ of one random digit, then the sum $S_2$ of two random digits, then the sum $S_3$ of three random digits and so on, and ask what is the probability that $n$ is a member of this sequence. We have one normal distribution for each $k$, and we want the total probability distribution that falls on $n$.
$endgroup$
– ShreevatsaR
Jan 7 at 3:57




$begingroup$
Thanks… (1) can you explain how you deal with the fact that the normal distribution is a continuous one and we're asking about the sum being exactly one $n$ (a point)? (2) My second question was about summing the above over all integers $k$. In other words, first we take the sum $S_1$ of one random digit, then the sum $S_2$ of two random digits, then the sum $S_3$ of three random digits and so on, and ask what is the probability that $n$ is a member of this sequence. We have one normal distribution for each $k$, and we want the total probability distribution that falls on $n$.
$endgroup$
– ShreevatsaR
Jan 7 at 3:57












$begingroup$
@ShreevatsaR: I expanded my answer in reply to your comment: does it meet your requirements ?
$endgroup$
– G Cab
Jan 7 at 17:21




$begingroup$
@ShreevatsaR: I expanded my answer in reply to your comment: does it meet your requirements ?
$endgroup$
– G Cab
Jan 7 at 17:21












$begingroup$
Thank you very much (+1). I'll think about this a bit more but I think the initial part matches. BTW for the second part, I was interested in the case where (in your notation) $m to infty$ -- note that as $k$ becomes larger than than some threshold, again the probability of hitting $n$ starts falling off exponentially, so it should be possible to sum over all $k$ and get an expression $Q(n; r)$ (in your notation) that depends only on $n$ (and $r$). Maybe not even $n$...
$endgroup$
– ShreevatsaR
Jan 7 at 18:36




$begingroup$
Thank you very much (+1). I'll think about this a bit more but I think the initial part matches. BTW for the second part, I was interested in the case where (in your notation) $m to infty$ -- note that as $k$ becomes larger than than some threshold, again the probability of hitting $n$ starts falling off exponentially, so it should be possible to sum over all $k$ and get an expression $Q(n; r)$ (in your notation) that depends only on $n$ (and $r$). Maybe not even $n$...
$endgroup$
– ShreevatsaR
Jan 7 at 18:36












$begingroup$
increasing $k$ increases the mean of the gaussian $p(n,r,k)$, while the std dev. increases with $sqrt{k}$. So, keeping $n$ fixed, the gaussian will move towards and then beyond it, with an $O(exp(-k)/ sqrt{k})$ which is certainly summable.
$endgroup$
– G Cab
Jan 7 at 19:22




$begingroup$
increasing $k$ increases the mean of the gaussian $p(n,r,k)$, while the std dev. increases with $sqrt{k}$. So, keeping $n$ fixed, the gaussian will move towards and then beyond it, with an $O(exp(-k)/ sqrt{k})$ which is certainly summable.
$endgroup$
– G Cab
Jan 7 at 19:22












$begingroup$
Yes exactly; that's what I expect and that's why I'd like to know asymptotically what the sum is equal to. Maybe I'll post a separate question about how to sum it over all $k$. :-) (and accept this one after a while).
$endgroup$
– ShreevatsaR
Jan 7 at 20:25




$begingroup$
Yes exactly; that's what I expect and that's why I'd like to know asymptotically what the sum is equal to. Maybe I'll post a separate question about how to sum it over all $k$. :-) (and accept this one after a while).
$endgroup$
– ShreevatsaR
Jan 7 at 20:25











1












$begingroup$

$newcommand{bbx}[1]{,bbox[15px,border:1px groove navy]{displaystyle{#1}},}
newcommand{braces}[1]{leftlbrace,{#1},rightrbrace}
newcommand{bracks}[1]{leftlbrack,{#1},rightrbrack}
newcommand{dd}{mathrm{d}}
newcommand{ds}[1]{displaystyle{#1}}
newcommand{expo}[1]{,mathrm{e}^{#1},}
newcommand{ic}{mathrm{i}}
newcommand{mc}[1]{mathcal{#1}}
newcommand{mrm}[1]{mathrm{#1}}
newcommand{pars}[1]{left(,{#1},right)}
newcommand{partiald}[3]{frac{partial^{#1} #2}{partial #3^{#1}}}
newcommand{root}[2]{,sqrt[#1]{,{#2},},}
newcommand{totald}[3]{frac{mathrm{d}^{#1} #2}{mathrm{d} #3^{#1}}}
newcommand{verts}[1]{leftvert,{#1},rightvert}$





  • Let $X_{1},ldots,X_{k}$ each be random digits. That is, they are independent random variables each uniformly distributed over the finite set $braces{0, 1, 2, 3, 4, 5, 6, 7, 8, 9}$.

  • Let $S equiv X_{1} + cdots + X_{k}$.

  • Given some integer $n$, what is the probability that $S = n$ ?.




begin{align}
&mathbb{P}bracks{X_{1} + cdots + X_{k} = n} equiv
bbox[10px,#ffd]{sum_{x_{1} = 0}^{9}{1 over 10}cdots
sum_{x_{k} = 0}^{9}{1 over 10}
bracks{z^{n}}z^{x_{1} + cdots + x_{k}}}
\[5mm] = &
{1 over 10^{k}}bracks{z^{n}}pars{sum_{x = 0}^{9}z^{x}}^{k} = {1 over 10^{k}}bracks{z^{n}}pars{z^{10} - 1 over z - 1}^{k}
\[5mm] = &
{1 over 10^{k}}bracks{z^{n}}
pars{1 - z^{10}}^{k}pars{1 - z}^{-k}
\[5mm] = &
{1 over 10^{k}}bracks{z^{n}}
bracks{sum_{ell = 0}^{k}{k choose ell}pars{-z^{10}}^{ell}}
bracks{sum_{m = 0}^{infty}{-k choose m}pars{-z}^{m}}
\[5mm] = &
{1 over 10^{k}}
sum_{ell = 0}^{k}{k choose ell}sum_{m = 0}^{infty}
bracks{{k + m - 1 choose m}pars{-1}^{m}}pars{-1}^{ell + m}
bracks{10ell + m = n}
\[5mm] = &
{1 over 10^{k}}
sum_{ell = 0}^{k}{k choose ell}pars{-1}^{ell}
sum_{m = 0}^{infty}{k + m - 1 choose k - 1}
bracks{m = n - 10ell}
\[5mm] = &
{1 over 10^{k}}
sum_{ell = 0}^{k}{k choose ell}pars{-1}^{ell}
{k + n - 10ell - 1 choose k - 1}bracks{n - 10ell geq 0}
\[5mm] = &
bbx{{1 over 10^{k}}
sum_{ell = 0}^{color{red}{M}}{k choose ell}
{k + n - 1 - 10ell choose k - 1}pars{-1}^{ell},,quad
color{red}{M} equiv
minbraces{k,leftlfloor{n over 10}rightrfloor}}
end{align}






share|cite|improve this answer











$endgroup$









  • 1




    $begingroup$
    This is already mentioned in the question (after the list of 19 other questions on this site). It's not useful because I want an asymptotic expression for large $n$ and $k$.
    $endgroup$
    – ShreevatsaR
    Jan 6 at 21:21










  • $begingroup$
    But thanks for taking the effort of typing this up, and please let me know how I could have written the question more clearly to avoid this — maybe assume that not everyone will read the whole question, and move this part to the top?
    $endgroup$
    – ShreevatsaR
    Jan 6 at 21:25










  • $begingroup$
    I've moved the formula from the middle to the top of the question — sorry for wasting your time; it didn't occur to me that someone might not read the whole question and end up writing something that's already mentioned in the question and in the answers to several other questions. Hope it's better now.
    $endgroup$
    – ShreevatsaR
    Jan 6 at 22:05










  • $begingroup$
    @ShreevatsaR Don't worry about it. It's fine you recognize my answer as useful one. Thanks.
    $endgroup$
    – Felix Marin
    Jan 7 at 17:41
















1












$begingroup$

$newcommand{bbx}[1]{,bbox[15px,border:1px groove navy]{displaystyle{#1}},}
newcommand{braces}[1]{leftlbrace,{#1},rightrbrace}
newcommand{bracks}[1]{leftlbrack,{#1},rightrbrack}
newcommand{dd}{mathrm{d}}
newcommand{ds}[1]{displaystyle{#1}}
newcommand{expo}[1]{,mathrm{e}^{#1},}
newcommand{ic}{mathrm{i}}
newcommand{mc}[1]{mathcal{#1}}
newcommand{mrm}[1]{mathrm{#1}}
newcommand{pars}[1]{left(,{#1},right)}
newcommand{partiald}[3]{frac{partial^{#1} #2}{partial #3^{#1}}}
newcommand{root}[2]{,sqrt[#1]{,{#2},},}
newcommand{totald}[3]{frac{mathrm{d}^{#1} #2}{mathrm{d} #3^{#1}}}
newcommand{verts}[1]{leftvert,{#1},rightvert}$





  • Let $X_{1},ldots,X_{k}$ each be random digits. That is, they are independent random variables each uniformly distributed over the finite set $braces{0, 1, 2, 3, 4, 5, 6, 7, 8, 9}$.

  • Let $S equiv X_{1} + cdots + X_{k}$.

  • Given some integer $n$, what is the probability that $S = n$ ?.




begin{align}
&mathbb{P}bracks{X_{1} + cdots + X_{k} = n} equiv
bbox[10px,#ffd]{sum_{x_{1} = 0}^{9}{1 over 10}cdots
sum_{x_{k} = 0}^{9}{1 over 10}
bracks{z^{n}}z^{x_{1} + cdots + x_{k}}}
\[5mm] = &
{1 over 10^{k}}bracks{z^{n}}pars{sum_{x = 0}^{9}z^{x}}^{k} = {1 over 10^{k}}bracks{z^{n}}pars{z^{10} - 1 over z - 1}^{k}
\[5mm] = &
{1 over 10^{k}}bracks{z^{n}}
pars{1 - z^{10}}^{k}pars{1 - z}^{-k}
\[5mm] = &
{1 over 10^{k}}bracks{z^{n}}
bracks{sum_{ell = 0}^{k}{k choose ell}pars{-z^{10}}^{ell}}
bracks{sum_{m = 0}^{infty}{-k choose m}pars{-z}^{m}}
\[5mm] = &
{1 over 10^{k}}
sum_{ell = 0}^{k}{k choose ell}sum_{m = 0}^{infty}
bracks{{k + m - 1 choose m}pars{-1}^{m}}pars{-1}^{ell + m}
bracks{10ell + m = n}
\[5mm] = &
{1 over 10^{k}}
sum_{ell = 0}^{k}{k choose ell}pars{-1}^{ell}
sum_{m = 0}^{infty}{k + m - 1 choose k - 1}
bracks{m = n - 10ell}
\[5mm] = &
{1 over 10^{k}}
sum_{ell = 0}^{k}{k choose ell}pars{-1}^{ell}
{k + n - 10ell - 1 choose k - 1}bracks{n - 10ell geq 0}
\[5mm] = &
bbx{{1 over 10^{k}}
sum_{ell = 0}^{color{red}{M}}{k choose ell}
{k + n - 1 - 10ell choose k - 1}pars{-1}^{ell},,quad
color{red}{M} equiv
minbraces{k,leftlfloor{n over 10}rightrfloor}}
end{align}






share|cite|improve this answer











$endgroup$









  • 1




    $begingroup$
    This is already mentioned in the question (after the list of 19 other questions on this site). It's not useful because I want an asymptotic expression for large $n$ and $k$.
    $endgroup$
    – ShreevatsaR
    Jan 6 at 21:21










  • $begingroup$
    But thanks for taking the effort of typing this up, and please let me know how I could have written the question more clearly to avoid this — maybe assume that not everyone will read the whole question, and move this part to the top?
    $endgroup$
    – ShreevatsaR
    Jan 6 at 21:25










  • $begingroup$
    I've moved the formula from the middle to the top of the question — sorry for wasting your time; it didn't occur to me that someone might not read the whole question and end up writing something that's already mentioned in the question and in the answers to several other questions. Hope it's better now.
    $endgroup$
    – ShreevatsaR
    Jan 6 at 22:05










  • $begingroup$
    @ShreevatsaR Don't worry about it. It's fine you recognize my answer as useful one. Thanks.
    $endgroup$
    – Felix Marin
    Jan 7 at 17:41














1












1








1





$begingroup$

$newcommand{bbx}[1]{,bbox[15px,border:1px groove navy]{displaystyle{#1}},}
newcommand{braces}[1]{leftlbrace,{#1},rightrbrace}
newcommand{bracks}[1]{leftlbrack,{#1},rightrbrack}
newcommand{dd}{mathrm{d}}
newcommand{ds}[1]{displaystyle{#1}}
newcommand{expo}[1]{,mathrm{e}^{#1},}
newcommand{ic}{mathrm{i}}
newcommand{mc}[1]{mathcal{#1}}
newcommand{mrm}[1]{mathrm{#1}}
newcommand{pars}[1]{left(,{#1},right)}
newcommand{partiald}[3]{frac{partial^{#1} #2}{partial #3^{#1}}}
newcommand{root}[2]{,sqrt[#1]{,{#2},},}
newcommand{totald}[3]{frac{mathrm{d}^{#1} #2}{mathrm{d} #3^{#1}}}
newcommand{verts}[1]{leftvert,{#1},rightvert}$





  • Let $X_{1},ldots,X_{k}$ each be random digits. That is, they are independent random variables each uniformly distributed over the finite set $braces{0, 1, 2, 3, 4, 5, 6, 7, 8, 9}$.

  • Let $S equiv X_{1} + cdots + X_{k}$.

  • Given some integer $n$, what is the probability that $S = n$ ?.




begin{align}
&mathbb{P}bracks{X_{1} + cdots + X_{k} = n} equiv
bbox[10px,#ffd]{sum_{x_{1} = 0}^{9}{1 over 10}cdots
sum_{x_{k} = 0}^{9}{1 over 10}
bracks{z^{n}}z^{x_{1} + cdots + x_{k}}}
\[5mm] = &
{1 over 10^{k}}bracks{z^{n}}pars{sum_{x = 0}^{9}z^{x}}^{k} = {1 over 10^{k}}bracks{z^{n}}pars{z^{10} - 1 over z - 1}^{k}
\[5mm] = &
{1 over 10^{k}}bracks{z^{n}}
pars{1 - z^{10}}^{k}pars{1 - z}^{-k}
\[5mm] = &
{1 over 10^{k}}bracks{z^{n}}
bracks{sum_{ell = 0}^{k}{k choose ell}pars{-z^{10}}^{ell}}
bracks{sum_{m = 0}^{infty}{-k choose m}pars{-z}^{m}}
\[5mm] = &
{1 over 10^{k}}
sum_{ell = 0}^{k}{k choose ell}sum_{m = 0}^{infty}
bracks{{k + m - 1 choose m}pars{-1}^{m}}pars{-1}^{ell + m}
bracks{10ell + m = n}
\[5mm] = &
{1 over 10^{k}}
sum_{ell = 0}^{k}{k choose ell}pars{-1}^{ell}
sum_{m = 0}^{infty}{k + m - 1 choose k - 1}
bracks{m = n - 10ell}
\[5mm] = &
{1 over 10^{k}}
sum_{ell = 0}^{k}{k choose ell}pars{-1}^{ell}
{k + n - 10ell - 1 choose k - 1}bracks{n - 10ell geq 0}
\[5mm] = &
bbx{{1 over 10^{k}}
sum_{ell = 0}^{color{red}{M}}{k choose ell}
{k + n - 1 - 10ell choose k - 1}pars{-1}^{ell},,quad
color{red}{M} equiv
minbraces{k,leftlfloor{n over 10}rightrfloor}}
end{align}






share|cite|improve this answer











$endgroup$



$newcommand{bbx}[1]{,bbox[15px,border:1px groove navy]{displaystyle{#1}},}
newcommand{braces}[1]{leftlbrace,{#1},rightrbrace}
newcommand{bracks}[1]{leftlbrack,{#1},rightrbrack}
newcommand{dd}{mathrm{d}}
newcommand{ds}[1]{displaystyle{#1}}
newcommand{expo}[1]{,mathrm{e}^{#1},}
newcommand{ic}{mathrm{i}}
newcommand{mc}[1]{mathcal{#1}}
newcommand{mrm}[1]{mathrm{#1}}
newcommand{pars}[1]{left(,{#1},right)}
newcommand{partiald}[3]{frac{partial^{#1} #2}{partial #3^{#1}}}
newcommand{root}[2]{,sqrt[#1]{,{#2},},}
newcommand{totald}[3]{frac{mathrm{d}^{#1} #2}{mathrm{d} #3^{#1}}}
newcommand{verts}[1]{leftvert,{#1},rightvert}$





  • Let $X_{1},ldots,X_{k}$ each be random digits. That is, they are independent random variables each uniformly distributed over the finite set $braces{0, 1, 2, 3, 4, 5, 6, 7, 8, 9}$.

  • Let $S equiv X_{1} + cdots + X_{k}$.

  • Given some integer $n$, what is the probability that $S = n$ ?.




begin{align}
&mathbb{P}bracks{X_{1} + cdots + X_{k} = n} equiv
bbox[10px,#ffd]{sum_{x_{1} = 0}^{9}{1 over 10}cdots
sum_{x_{k} = 0}^{9}{1 over 10}
bracks{z^{n}}z^{x_{1} + cdots + x_{k}}}
\[5mm] = &
{1 over 10^{k}}bracks{z^{n}}pars{sum_{x = 0}^{9}z^{x}}^{k} = {1 over 10^{k}}bracks{z^{n}}pars{z^{10} - 1 over z - 1}^{k}
\[5mm] = &
{1 over 10^{k}}bracks{z^{n}}
pars{1 - z^{10}}^{k}pars{1 - z}^{-k}
\[5mm] = &
{1 over 10^{k}}bracks{z^{n}}
bracks{sum_{ell = 0}^{k}{k choose ell}pars{-z^{10}}^{ell}}
bracks{sum_{m = 0}^{infty}{-k choose m}pars{-z}^{m}}
\[5mm] = &
{1 over 10^{k}}
sum_{ell = 0}^{k}{k choose ell}sum_{m = 0}^{infty}
bracks{{k + m - 1 choose m}pars{-1}^{m}}pars{-1}^{ell + m}
bracks{10ell + m = n}
\[5mm] = &
{1 over 10^{k}}
sum_{ell = 0}^{k}{k choose ell}pars{-1}^{ell}
sum_{m = 0}^{infty}{k + m - 1 choose k - 1}
bracks{m = n - 10ell}
\[5mm] = &
{1 over 10^{k}}
sum_{ell = 0}^{k}{k choose ell}pars{-1}^{ell}
{k + n - 10ell - 1 choose k - 1}bracks{n - 10ell geq 0}
\[5mm] = &
bbx{{1 over 10^{k}}
sum_{ell = 0}^{color{red}{M}}{k choose ell}
{k + n - 1 - 10ell choose k - 1}pars{-1}^{ell},,quad
color{red}{M} equiv
minbraces{k,leftlfloor{n over 10}rightrfloor}}
end{align}







share|cite|improve this answer














share|cite|improve this answer



share|cite|improve this answer








edited Jan 6 at 21:22

























answered Jan 6 at 20:40









Felix MarinFelix Marin

68.9k7110147




68.9k7110147








  • 1




    $begingroup$
    This is already mentioned in the question (after the list of 19 other questions on this site). It's not useful because I want an asymptotic expression for large $n$ and $k$.
    $endgroup$
    – ShreevatsaR
    Jan 6 at 21:21










  • $begingroup$
    But thanks for taking the effort of typing this up, and please let me know how I could have written the question more clearly to avoid this — maybe assume that not everyone will read the whole question, and move this part to the top?
    $endgroup$
    – ShreevatsaR
    Jan 6 at 21:25










  • $begingroup$
    I've moved the formula from the middle to the top of the question — sorry for wasting your time; it didn't occur to me that someone might not read the whole question and end up writing something that's already mentioned in the question and in the answers to several other questions. Hope it's better now.
    $endgroup$
    – ShreevatsaR
    Jan 6 at 22:05










  • $begingroup$
    @ShreevatsaR Don't worry about it. It's fine you recognize my answer as useful one. Thanks.
    $endgroup$
    – Felix Marin
    Jan 7 at 17:41














  • 1




    $begingroup$
    This is already mentioned in the question (after the list of 19 other questions on this site). It's not useful because I want an asymptotic expression for large $n$ and $k$.
    $endgroup$
    – ShreevatsaR
    Jan 6 at 21:21










  • $begingroup$
    But thanks for taking the effort of typing this up, and please let me know how I could have written the question more clearly to avoid this — maybe assume that not everyone will read the whole question, and move this part to the top?
    $endgroup$
    – ShreevatsaR
    Jan 6 at 21:25










  • $begingroup$
    I've moved the formula from the middle to the top of the question — sorry for wasting your time; it didn't occur to me that someone might not read the whole question and end up writing something that's already mentioned in the question and in the answers to several other questions. Hope it's better now.
    $endgroup$
    – ShreevatsaR
    Jan 6 at 22:05










  • $begingroup$
    @ShreevatsaR Don't worry about it. It's fine you recognize my answer as useful one. Thanks.
    $endgroup$
    – Felix Marin
    Jan 7 at 17:41








1




1




$begingroup$
This is already mentioned in the question (after the list of 19 other questions on this site). It's not useful because I want an asymptotic expression for large $n$ and $k$.
$endgroup$
– ShreevatsaR
Jan 6 at 21:21




$begingroup$
This is already mentioned in the question (after the list of 19 other questions on this site). It's not useful because I want an asymptotic expression for large $n$ and $k$.
$endgroup$
– ShreevatsaR
Jan 6 at 21:21












$begingroup$
But thanks for taking the effort of typing this up, and please let me know how I could have written the question more clearly to avoid this — maybe assume that not everyone will read the whole question, and move this part to the top?
$endgroup$
– ShreevatsaR
Jan 6 at 21:25




$begingroup$
But thanks for taking the effort of typing this up, and please let me know how I could have written the question more clearly to avoid this — maybe assume that not everyone will read the whole question, and move this part to the top?
$endgroup$
– ShreevatsaR
Jan 6 at 21:25












$begingroup$
I've moved the formula from the middle to the top of the question — sorry for wasting your time; it didn't occur to me that someone might not read the whole question and end up writing something that's already mentioned in the question and in the answers to several other questions. Hope it's better now.
$endgroup$
– ShreevatsaR
Jan 6 at 22:05




$begingroup$
I've moved the formula from the middle to the top of the question — sorry for wasting your time; it didn't occur to me that someone might not read the whole question and end up writing something that's already mentioned in the question and in the answers to several other questions. Hope it's better now.
$endgroup$
– ShreevatsaR
Jan 6 at 22:05












$begingroup$
@ShreevatsaR Don't worry about it. It's fine you recognize my answer as useful one. Thanks.
$endgroup$
– Felix Marin
Jan 7 at 17:41




$begingroup$
@ShreevatsaR Don't worry about it. It's fine you recognize my answer as useful one. Thanks.
$endgroup$
– Felix Marin
Jan 7 at 17:41


















draft saved

draft discarded




















































Thanks for contributing an answer to Mathematics Stack Exchange!


  • Please be sure to answer the question. Provide details and share your research!

But avoid



  • Asking for help, clarification, or responding to other answers.

  • Making statements based on opinion; back them up with references or personal experience.


Use MathJax to format equations. MathJax reference.


To learn more, see our tips on writing great answers.




draft saved


draft discarded














StackExchange.ready(
function () {
StackExchange.openid.initPostLogin('.new-post-login', 'https%3a%2f%2fmath.stackexchange.com%2fquestions%2f3063646%2fwhat-is-the-probability-that-the-sum-of-digits-of-a-random-k-digit-number-is%23new-answer', 'question_page');
}
);

Post as a guest















Required, but never shown





















































Required, but never shown














Required, but never shown












Required, but never shown







Required, but never shown

































Required, but never shown














Required, but never shown












Required, but never shown







Required, but never shown







Popular posts from this blog

How do I know what Microsoft account the skydrive app is syncing to?

Grease: Live!

When does type information flow backwards in C++?